Chapter 72: Emergency Nursing

Lakukan tugas rumah & ujian kamu dengan baik sekarang menggunakan Quizwiz!

A comprehensive, valid, and reliable five level triage

Severity rating system that recognizes that EDs are used for both emergency and routine health care

To limit the risk of exposure to airborne diseases,

early identification and strict adherence to transmission based precautions for pts who are potentially infectious is crucial

Decompression sickness

A painful condition seen in divers who ascend too quickly, in which gas, especially nitrogen, forms bubbles in blood vessels and other tissues; also called "the bends." More significantly nitrogen bubbles can become air emboli in the bloodstream leading to stroke, paralysis, or death Musculoskeletal pain Numbness Hypesthesia

The goal of triage is

A rapid assessment and decision making preferably under 5 mins

Rhabdomyolysis

A toxic syndrome caused by widespread injury to skeletal muscle, resulting in myoglobinuria and acute kidney injury

Which statement reflects the nursing management of the client with a white phosphorus chemical burn? A. Do not apply water to the burn B. Wash off the chemical using warm water, then flush the skin with cool water C. Alternate applications of water and ice to the burn D. Immediately drench the skin with running water from a shower, hose, or faucet

A. Do not apply water to the burn Water should not be applied to burns from lye or white phosphorus because of the potential for an explosion or deepening of the burn.

Assessment and Diagnostic findings for airway obstruction

Ask the person if they are choking and need help If the person is unconscious - inspection of the oropharynx may reveal the offending object X rays, laryngoscopy, or bronchoscopy Oxygen supplementation immediately

Causes of upper airway obstruction

Aspiration of foreign objects Anaphylaxis Viral or bacterial infection Trauma Inhalation or chemical burns

Maintaining ventilation

Assess for absent or diminished breath sounds, open chest wounds, and difficulty delivering artificial breaths for the pt Monitor pulse oximetry, capnography, and arterial blood gases if the pt requires airway or ventilatory assistance

Following a motor vehicle collision, a client is brought to the ED for evaluation and treatment. The client is being assessed for intra-abdominal injuries. The client reports severe left shoulder pain (pain score of 10 on a 1 to 10 scale). The nurse suspects injury to the A. gallbladder. B. large intestine. C. spleen. D. liver.

C. spleen. The location of pain can indicate certain types of intra-abdominal injuries. Pain in the left shoulder is common in a client with bleeding from a ruptured spleen, whereas pain in the right shoulder can result from laceration of the liver.

Partial airway occlusion

Can lead to progressive hypoxia, hypercabia, and respiratory and cardiac arrest

Any small bleeding vessels are

Clamped, tied, or cauterized

Management of decompression sickness

Need rapid transfer to a hyperbaric chamber 100% oxygen Chest x-ray IV fluids (isotonic fluids) Low altitude flight below 300m Head of bed should be lowered if an air embolus is suspected Antibiotics if aspiration is suspected

Hematoma

Tumor like mass of blood trapped under the skin

Packed red blood cells are infused when

There is a massive blood loss, which may also necessitate transfusion of the blood components, including platelets and clotting factors

Emergent category

These pts must be reassessed at least every 15mins

Urgent category

These pts must be reassessed at least every 30 mins

Resuscitation category

These pts must receive continuous nursing surveillance

Cricothyroidotomy

This procedure is used in emergency situations in which endotracheal intubation is either not possible or contraindicated, as in airway obstruction from extensive maxillofacial trauma, cervical spine injuries, laryngospasm, laryngeal edema (after an allergic reaction or extubation), hemorrhage into neck tissue, or obstruction of the larynx It is placed with a formal tracheostomy when the pt is able to tolerate this procedure

People at risk for exertional heat stroke

Those not acclimatized to heat Those who are older or very young Those unable to care for themselves Those with chronic and debilitating diseases Those taking certain meds like major tranquilizers, anticholinergics, diuretics, beta blockers

Abdominal ultrasound can be used

To rapidly assess pts who are hemodynamically unstable to detect intraperitoneal bleeding This is referred to as focused assessment with sonography for trauma (FAST) examination

Minor bleeding which is usually venous generally stops spontaneously

Unless the pt has a bleeding disorder or has been taking anticoagulant agents

Defusing occurs

Immediately after a critical incident Affected staff are encouraged to discuss their feelings about the incident and are given contact information so that they may talk to someone if they have disturbing symptoms (sleeplessness, excessive worry)

Nursing diagnoses for establishing an airway

Ineffective airway clearance related to obstruction of the airway by the tongue, an object, or fluids ( blood or saliva) Ineffective breathing pattern related to airway obstruction or injury

The pt needs to give consent for

Invasive procedures (eg angiography, lumbar puncture) unless he or she is unconscious or in a critical condition and unable to make decisions

A triage nurse in the ED determines that a patient with dyspnea and dehydration is not in a life-threatening situation. What triage category will the nurse choose? A. Delayed B. Emergent C. Urgent D. Immediate

C. Urgent A basic and widely used triage system that had been in use for many years utilized three categories: emergent, urgent, and nonurgent. In this system, emergent patients had the highest priority, urgent patients had serious health problems but not immediately life-threatening ones, and nonurgent patients had episodic illnesses.

According to the Emergency Medical Treatment and Active Labor Act (EMTALA):

Every ED with a Medicare provider agreement must perform a medical screening examination on all pts arriving with an emergency medical complaint if their acute and symptoms could result in serious injury or death if left untreated

Fasciotomy is performed by a physician if

Compartment syndrome develops

A tetanus booster must be given if

The pt's last tetanus booster was given more than 5yrs ago

Critical incident stress management (CISM) may consist three steps

Defusing Debriefing Follow up

Abrasion

Denuded skin

Nonurgent category

Pts must be reassessed at least every 120 mins

Avulsion

Tearing away of tissue from supporting structures

Remorse and guilt

Stage when family members accuse themselves of negligence or minor omissions Encourage family members to verbalize their feelings to help them cope appropriately

The triage nurse is also responsible for

Monitoring the waiting area Maintaining a safe environment Reassessing pts who are waiting Is the initial liaison to the families of pts

Penetrating abdominal injuries (gunshot wounds, stab wounds)

are serious and usually require surgery Results in a high incidence of injury to hollow organs particularly the small bowel The liver is the most injured solid organ because of its placement and size All abd gunshot wounds that cross the peritoneum or are associated peritoneal signs require surgical exploration

Envenomation

injection of a poisonous material by sting, spine, bite, or other means

Heat stroke

Most serious Is an acute medical emergency caused by failure of heat regulating mechanisms of the body Inability to maintain cardiac output in the face of moderately high body temperatures and is associated with dehydration Most common cause of heat stroke- nonexertional, prolonged exposure to an environmental temperature of greater than 39.2C(102.5F) although a heat index of greater than 35C is associated with increased mortality

Cardinal manifestations of heat cramps

Muscle cramps particularly in the shoulders, abdomen, and lower extremities Profound diaphoresis Profound thirst

The classic triad of clinical manifestations of suggestive of rhabdomyolysis includes

Myalgias(muscle cramps) Generalized weakness Dark urine

Corrosive poison

Alkaline or acidic agent; causes tissue destruction after contact

Most common causes of airway obstruction are from

An allergic reaction or angioedema

Primary survey

An assessment of the pt triaged to emergent or resuscitation category that focuses on stabilizing life threatening conditions; uses the mnemonic ABCDE, which stands for airway, breathing, circulation, disability, and exposure

Serum lactic acid decrease to less than 2.5mmol/L is

An indication of successful resuscitation

Sentinel event

An unanticipated event that results in pt harm

Trauma

An unintentional or intentional wound or injury inflicted on the body from a mechanism against which the body cannot protect itself is the fourth leading cause in the United States of death

Possible nursing diagnoses include

Anxiety or death anxiety related to uncertain potential outcomes of the illness or trauma Ineffective coping related to acute situational crisis

Triage consists of providing basic first aid

Application of ice Bleeding control Basic wound care Initiating protocol based prescriptions (eg X-rays, administration of antipyretics or mild analgesic agents Obtaining an electrocardiogram (ECG), urinalysis or removing sutures

Quality and safety nursing alert

*THE LOCATION OF PAIN CAN INDICATE CERTAIN TYPES OF INTRAABDOMINAL INJURIES. PAIN IN THE LEFT SHOULDER IS COMMON IN A PT WITH A RUPTURED SPLEEN, WHEREAS PAIN IN THE RIGHT SHOULDER CAN RESULT FROM LACERATION OF THE LIVER**

Preventing heat induced illnesses

-Advise the pt to avoid immediate re-exposure to high temperatures -Emphasize the importance of maintaining adequate fluid intake, wearing loose clothing, and reducing activity in hot weather -Advise athletes to monitor fluid losses and weight loss during workout activities and to replace these fluids/electrolytes -Advise the patient to use a gradual approach to physical conditioning -Direct older adults to A/C -Advise patients to avoid outdoor activity between 10-2 (hottest time of day)

Supportive care during rewarming

-External *cardiac compressions* in pt with temps higher than 88F -*Defib* of v-fib (ineffective if <88 - must be rewarmed first) -*Mechanical ventilation with positive end expiratory pressure (PEEP) and heated humidified oxygen to maintain tissue oxygenation -*Warmed IV fluids* -*Sodium bicarbonate* -*Antiarrhythmia* meds -Insertion of *Indwelling urinary catheter* to monitor urinary output and kidney function

After inserting an oropharyngeal airway, the nurse determines that it is in the proper position when the flange is located at which position? A. Approximately at the patient's lips B. At the level of the patient's epiglottis C. Directly in front of the patient's teeth D. Just below the tip of the patient's nose

A. Approximately at the patient's lips When an oropharyngeal airway is properly inserted, the tip is in the hypopharynx and the flange is approximately at the patient's lips.

An 85-year-old client is admitted to the ED. Heat stroke is suspected. The client's core temperature is 106.2°F (41.2°C), blood pressure (BP) 90/60 mm Hg, and pulse 102 bpm. The nurse understands that the primary treatment measure for the client will include A. immersion of the client in a cold-water bath. B. IV hydration with normal saline solution. C. administration of sodium supplements. D. endotracheal intubation with mechanical ventilation.

A. immersion of the client in a cold-water bath. For the client with heat stroke, simultaneous treatment focuses on stabilizing oxygenation using the CABs (circulation, airway, and breathing; formerly called the ABCs) of basic life support. This includes establishing IV access for fluid administration. After the client's clothing is removed, the core (internal) temperature is reduced to 39°C (102°F) as rapidly as possible, preferably within 1 hour. One or more of the following methods may be used as prescribed: cool sheets and towels or continuous sponging with cool water; ice applied to the neck, groin, chest, and axillae while spraying with tepid water; and cooling blankets. Immersion of the client in a cold-water bath is the optimal method for cooling (if available). Hydration would be with lactated Ringer solution. There is no indication for intubation. Administration of sodium supplements is indicated for the treatment of heat cramps.

If a person survives submersion

Acute respiratory distress syndrome (ARDS) resulting in hypoxia, hypercarbia, and respiratory or metabolic acidosis can occur

A client presents to the ED with serious health problems that are not immediately life threatening. The nurse will correctly triage the client into which category? A. Psychological support B. Emergent C. Nonurgent D. Urgent

D. Urgent Clients triaged as urgent have serious health problems that are not immediately life threatening. They must be seen within 1 hour. The emergent category is for clients who have the highest priority conditions that are life-threatening and they must be seen immediately. Nonurgent is for clients who have episodic illness that can be addressed within 24 hours without increased morbidity. Clients in the less urgent category must be reassessed at least every 60 minutes and do not have serious health problems.

Three components to injury prevention

Education Legislation - meant to provide universal safety without infringing on rights Automatic protection

Three categories of triage (basic)

Emergent Urgent Non urgent

Solutions to pt safety issues in the ED include

Ensuring optimal nurse staffing Pharmacy presence Rapid diagnostic turnaround times to minimize wait time Fostering team work Support by leadership

Non urgent pts

Episodic illnesses

Adequate ventilation is evidenced by

Equal bilateral breath sounds

Clinical manifestations of airway obstruction

Foreign body obstruction - cannot speak, breath, or cough. May clutch the neck between the thumb and fingers ( universal distress signal) Chocking Apprehensive appearance Refusing to lie flat Insipiratory and expiratory stridor Labored breathing The use of accessory muscles ( suprasternal and intercostal retraction) Flaring nostrils Increasing anxiety Restlessness Confusion **ACTION MUST BE TAKEN BEFORE THESE MANIFESTATIONS DEVELOP OR IMMEDIATELY WITH EXHIBITION OF THESE SIGNS**

Nurses in the triage area collect additional crucial baseline data

Full vital signs including pain assessment History of current event and past medical history Neurologic assessment findings Weight Allergies (especially to latex and medications) Domestic violence screening Necessary diagnostic data I

Possible nursing diagnoses for the family include

Grieving Interrupted family processes Compromised or disabled family coping related to acute situational crises

Management of snake bites

Have person lie down, removing constrictive items such as rings Provide warmth Cleanse the wound and cover with a light sterile dressing Immobilize the injured part below the level of the heart ABC Do not apply ice, incision, suction, or a tourniquet Tetanus and pain meds

Pt is assessed for neurovascular compromise

If pain or pressure is reported

Lab studies in intraabdominal injuries

Hct, Hgb levels Lactate to determine acidosis and need for continued resuscitation ABG for pH (acidosis) Base deficit for resuscitation evaluation Ventilation parameters (PaCO2, PaO2) INR to identify coagulopathy or presence of chemically induced anticoagulation WBC count to detect elevation (generally associated with trauma)

Completely obstructed airway

Leads to permanent brain injury or death within 3 to 5mins secondary to hypoxia Air movement is absent Oxygen saturation of the blood decreases rapidly because obstruction of the airway prevents entry of air into the lungs Oxygen deficit occurs in the brain, resulting in unconsciousness, with death following rapidly

Salt water aspiration

Leads to pulmonary edema from the osmotic effects of the salt within the lungs

Other causes of rhabdomyolysis

Major burns Heat stroke Abuse of illicit drugs

Crush injuries

Occur when a person is caught between opposing forces (run over by a vehicle, crushed between two cars, crushed under a collapsed building)

Intraabdominal injuries are categorized as

Penetrating or blunt trauma

Diagnostic procedures to determine if there is intraperitoneal injury or bleeding

Peritoneal lavage Abdominal ultrasonography Abd CT scanning

Other causes of airway obstruction

Peritonsillar abscesses Epiglottis Acute infections of the posterior pharynx

Triage

Process of assessing pts to determine management priorities

Manifestations of heat stroke

Profound CNS dysfunction (confusion, delirium, bizarre behavior, coma seizures) Elevated body temp 40.6C (105F) or higher Hot, dry skin and usually anhidrosis (absence of sweating) Tachypnea Hypotension Tachycardia

Signs of peritoneal irritation include

Progressive abdominal distention Involuntary guarding Tenderness Pain Muscular rigidity Rebound tenderness Changes in bowel sounds Hypotension and s/s of shock

A patient is hemorrhaging from an open wound on his leg. The nurse implements care using the following steps. Place them in the order in which the nurse would perform them. Use all options. 3Provide firm direct pressure 4Immobilize the leg 2Elevate the leg 1Apply a pressure dressing

Provide firm direct pressure Apply a pressure dressing Elevate the leg Immobilize the leg When a patient is hemorrhaging from a leg wound, first the nurse would apply direct firm pressure to control the bleeding. Next, the nurse would apply a pressure dressing, and elevate the injured area to stop venous and capillary bleeding if possible. Then, the area is immobilized to control blood loss.

The ED nurse must also focus on

Providing comfort and emotional support to the pt and family

Less urgent category

Pts must be reassessed at least every 60 mins

Management of hypothermia

Removal of wet clothing Continuous monitoring Supportive care

Resuscitation procedures

Restoration of airway, breathing, and circulation

Heat stroke causes thermal injury at the cellular level

Resulting in coagulopathies and widespread damage to the heart, liver, and kidneys

Fresh water aspiration

Results in a loss of surfactant, an inability to expand the lungs

Laceration

Skin tear with irregular edges and vein bridging

A tension pneumothorax can mimic hypovolemia

So ventilatory assessment precedes assessment for hemorrhage

Cricothyroidotomy

Surgical opening of the cricothyroid membrane to obtain an airway that is maintained with a tracheostomy or endotracheal tube

Nonfatal drowning

Survival for at least 24h after submersion that caused respiratory arrest Most common consequence is hypoxemia Children under 5 and those over 85 have the highest risk of drowning The nonfatal drowning process involves the onset of hypoxia, hypercapnia, bradycardia, and dysrhythmias

Primary closure (wound)

The decision to suture a wound depends on the nature of the wound, the time since the injury was sustained, the degree of contamination and the vascularity

Management of frost bite

The goal is to restore normal body temp Constrictive and wet clothes removed (more rapid removal of wet clothes) Pt should not be allowed to ambulated if lower extremities are involved Controlled yet rapid rewarming Frozen extremities are usually placed in a 37C to 40C (98.6 to 104F) circulating bath for 30 to 40 min spans - repeated until circulation is restored Early warming decreases the amount of ultimate tissue loss Pain meds given during the rewarming process as it may be very painful No handling or massing of affected body part Once rewarmed, the part is protected from further injury and is elevated to help control swelling Sterile gauze or cotton between fingers and toes to prevent mace ration Hemorrhagic blobs which may develop 1hr to a few days after rewarming are left intact and not ruptured Nonhemorrhagic blisters are debrided to decrease the inflammatory mediators found in the blister fluid Whirlpool bath for the affected body parts to aid circulation and debridement of necrotic tissue to help prevent infection Escharotomy (incision through the eschar) to prevent further tissue damage, to allow for normal circulation and permit joint motion Fasciotomy to treat compartment syndrome After rewarming, hrly active motion of any affected digits is encouraged to promote maximal restoration of function and to prevent contractures Pt to avoid tobacco, alcohol, and caffeine because of their vasoconstrictive effects, which further reduce the already deficient blood supply to injured tissue

Sinography

Used in stab wounds to detect peritoneal penetration

Snake bites

Venomous snake bites are a medical emergency

Most common type of violence

Verbal abuse

Patterned

Wound representing the outline of the object ( eg steering wheel) causing a wound

Fasciotomy

a surgical incision through the fascia to relieve tension or pressure and restore neurovascular function to the extremity

A patient in the emergency department is bleeding profusely from numerous large and deep lacerations on the top of his head, right side of his face, and forehead. The nurse determines the need to apply pressure at the appropriate pressure point. The nurse would use which of the following pressure points?

right next to the ear The location of the injuries and site of bleeding determine which pressure point to use. In this case, the patient's bleeding is proximal to the temporal artery; therefore, pressure should be applied to this area, as shown in option A. If the patient was bleeding from the lower portion of the face, pressure would be applied to the facial artery, as in option B. The carotid artery would be used to control bleeding proximal to that area. The subclavian artery would be used to control bleeding proximal to it, such as the lower neck and shoulder area.

Control of internal bleeding

Administration of packed red blood cells, plasma, and platelets at a rapid rate and the pt is prepared for more definitive treatment ( surgery or pharmacologic therapy) Obtain ABG specimen to evaluate pulmonary function and tissue perfusion and to establish baseline hemodynamic parameters to be used as an index for determining the amt of fluid replacement the pt can tolerate and the response to therapy The pt is maintained in a supine position and monitored closely until hemodynamic or circulatory parameters improve, or until he or she is transported to the operative room or intensive care unit

Caring for Emergency Personnel

After serious events, critical incident stress management (CISM) is necessary to critique individual and group performance and to facilitate healthy coping

Follow up may occur

After the debriefing session is completed for those participants who have persistent negative symptoms and may consist of continued individual or group counseling and therapy

Fractures

Assessment is conducted for pain over or near the bone, swelling (from blood, lymph, and exudate infiltrating the tissue), and circulatory disturbance Assessment of ecchymosis, tenderness, and crepitation

Ecchymosis/contusion

Blood trapped under the surface of the skin

A triage nurse determines that a client with non-life-threatening injuries requires imaging studies and moderate sedation. The triage nurse would document this client as which of the following? A. Resuscitation B. Nonurgent C. Urgent D. Emergent

C. Urgent Clients who have non-life-threatening conditions but require two or more resouces to provide their care would be classified as urgent. In this situation, the client would be considered urgent becuase he requires imaging studies (one resource) and moderate sedation (a two-resource procedure). Clients in the resuscitation category need treatment immediately to prevent death. Clients in the emergent category may deteriorate rapidly and develop a major life-threatening situation or may require time-sensitive treatment. Clients in the nonurgent category have non-life-threatening conditions and likely need only one resource to provide for their needs.

The nurse is caring for a client who is being prepared for the placement of a central intravenous line. The nurse recognizes this client requires this type of intravenous access for which reason? A. The client will require intravenous access for three days B. The client requires infusion of a dextrose 5% water (D5W) C. The client requires infusion of intravenous antibiotics D. The client requires total parenteral nutrition

D. The client requires total parenteral nutrition For a patient who requires total parenteral nutrition (TPN), a central intravenous line is required due to the length of time the client will require the infusion as well as the nature of the solution itself. A large vein is required to safely infuse TPN. For this reason, a central line is needed. A peripheral intravenous line is safe to used when IV access is required under six days. Beyond this time, either a new peripheral IV will need to be inserted. If it is known in advance that IV treatment will last beyond six days, the client's health care provider will order the placement of a central intravenous line. Intravenous antibiotics can be administered peripherally unless the course is longer than six days. D5W is an intravenous solution that can be administered either peripherally or centrally. The nature of this IV solution would not determine which type of IV access the client requires.

A nurse is providing initial first-aid care to a patient who was bitten by a snake. Place the following actions in the order in which the nurse would perform them. Use all options. 1Cover the wound with a light sterile dressing 2Provide warmth 3Remove constricting clothing 4Immobilize the injury below the level of the heart 5Have the patient lie down 6Clean the wound

Have the patient lie down Remove constricting clothing Provide warmth Clean the wound Cover the wound with a light sterile dressing Immobilize the injury below the level of the heart Initial first aid for a snake bite includes having the person lie down, removing constrictive items, providing warmth, cleaning the wound, covering the wound with a light and sterile dressing, and immobilizing the injured body part below the level of the heart.

Internal hemorrhage can hide in many anatomical spaces and compartments resulting

In shock without external evidence of hemorrhage

Cut

Incision of the skin with well defined edges usually long than deep

Stab

Incision of the skin with well defined edges, usually caused by a sharp instrument, a stab wound is typically deeper than long

Airway obstruction

Is a life threatening medical emergency

Referred pain is a significant finding because

It suggests intraperitoneal injury

Clinical manifestations of venomous snake bites

May affect neurologic, cardiovascular, and respiratory systems Classic clinical signs of envenomation - edema, ecchymosis, and hemorrhagic bullae leading to necrosis to the site of envenomation Symptoms - lymph node tenderness, nausea, vomiting, numbness, and metallic taste in the mouth Without treatment the clinical manifestations may progress to include fasciculations, hypotension, parasthesias, seizures, and coma

Establishing an Airway

May be as simple as repositioning the pt's head to prevent the tongue from obstructing the pharynx Head tilt/ chin lift maneuver Jaw thrust maneuver Insertion of specialized equipment may be needed to remove the foreign body or maintain airway **IN ALL MANEUVERS THE CERVICAL SPINE MUST BE PROTECTED FROM INJURY**

Delayed primary closure

May be indicated if tissue has been lost or there is a high potential for infection The wound is splinted in a functional position to prevent motion and decrease the possibility of contracture Wound may be sutured if no s/s of suppuration (formation of purulent drainage)

The nurse is preparing to transfer a client from the ICU to a medical unit in the hospital. To ensure consistent communication regarding the client's care needs to the receiving unit, in what sequence of steps should the nurse organize the report? 1Obtain the client's health record 2Give recommendations for what needs to be done for the client 3Provide the most recent vital signs and assessment findings 4Provide a brief statement of current concerns 5State the client's admission date and current diagnosis 6Give the client's pertinent medical history

Obtain the client's health record State the client's admission date and current diagnosis Provide a brief statement of current concerns Give the client's pertinent medical history Provide the most recent vital signs and assessment findings Give recommendations for what needs to be done for the client When using the SBAR tool for consistent communication in health care settings, the nurse should organize sharing information about the client by including what the receiving unit needs to know about the (S)ituation, (B)ackground, (A)ssessment and (R)ecommendations. The nurse should first have the chart in hand before making the phone call, and be sure they can readily communicate all the following: Briefly state the issue or problem: what it is, when it happened (or how it started) and how severe it is. Give the signs and symptoms that cause concern. The nurse should then provide the date of admission and current medical diagnoses. Next, the nurse must give most recent vital signs and any recent changes in the systems assessment. For example, the nurse may need to communicate that the client had become constipated over the past 24 hours. Finally, it is important for the nurse to provide recommendations about what actions are need to be taken in the client's care. The nurse should state what they think should be done to address any identified client problems.

The nurse is providing care for a client who was admitted to the intensive care unit after suffering cardiovascular collapse secondary to a methamphetamine overdose. The client is semi-conscious and has a nasopharyngeal in place. The nurse anticipates this client may require which interventions? Select all that apply. A. Minimize lights and noise disturbances B. Administer antipsychotic medication C. Provide airway support and ventilation D. Follow the unit seizure protocol E. Apply warming blankets

Provide airway support and ventilation Minimize lights and noise disturbances Administer antipsychotic medication Follow the unit seizure protocol Anticipated interventions for client who have experienced cardiovascular collapse secondary to overdose with methamphetamines include prioritizing airway support, ventilation, cardiac monitoring and intravenous access. The nurse should attempt to provide a calm environment that is as private as possible. Lights and noise disturbances should be kept to a minimum because external stimulation can produce overactivity and overstimulation. Due to the hallucinations and/or delusions that can be caused by the illicit substance overdose, the client may require antipsychotic medication such as haloperidol. Clients can experience seizures after illicit substance overdose and withdrawal. The nurse should anticipate the need to employ a seizure protocol in accordance with unit policy. Clients with methamphetamine overdoses experience hypertension and hyperthermia. Warming the client potentiates amphetamine toxicity.

A key principle underlying emergency care is that

The pt is rapidly assessed, treated, and referred to the appropriate setting for ongoing care

The internal spaces and compartments that are capable of housing large amts of blood include the

Retroperitoneum Pelvis Chest Thighs

Severe muscular damage may cause

Rhabdomyolysis which leads to (ATN)acute tubular necrosis

Urgent patients

Serious health problems but not immediately life threatening ones

Most sensitive indicator of rhabdomyolysis

Serum Creatine kinase (CK) - levels in excess of 6000IU/L are considered diagnostic

Heat exhaustion or heat cramps

Should lie supine in a cool environment Heat exhaustion- IV fluids or oral fluids if tolerated Heat cramps - oral sodium supplements, oral electrolyte solutions

Pressure points for hemorrhage control

Temporal Facial Carotid Subclavian Brachial Radial and ulnar Femoral

The nurse is monitoring a client who is receiving a transfusion of packed red blood cells (PRBCs) for impaired tissue oxygenation resulting from hemorrhage. After 15 minutes of the transfusion, the nurse notes the client has a fever and shortness of breath. Place in order the steps the nurse should take in response to these findings. Use all options. 3Stop the transfusion 6Assess need for airway support 2Check full vital signs 4Intervene for any signs and symptoms as appropriate 5Ensure the normal saline IV line is open 1Notify the physician

Stop the transfusion Ensure the normal saline IV line is open Assess need for airway support Check full vital signs Notify the physician Intervene for any signs and symptoms as appropriate The client is experiencing an immunological transfusion reaction which will only become worse as the transfusion proceeds. The nurse's first action is to stop the transfusion. A normal saline line is always made available prior to commencing a transfusion of any blood product to promote flushing and allowing for the immediate administration of any IV medications that may be required to manage the signs and symptoms resulting from the transfusion reaction. Airway, circulation and breathing are a top priority in transfusion reactions. The client is experiencing shortness of breath which can progress to respiratory distress if not managed. The nurse must assess airway and work of breathing to determine if oxygen, repositioning or other respiratory interventions are required. The nurse must assess a full set of vital signs to determine other systemic effects caused by the transfusion. It is possible to see variations of vital signs such as hyper- and hypotension, tachycardia, fever and increased respiratory rate. Any change in the vital signs requires an intervention. This should be completed prior to contacting the physician as it is important to have this information readily available to collaborate with the physician for next steps in the client's care. The nurse must notify the physician to obtain any additional orders for interventions that may be individualized based on the client's overall clinical situation. The nurse is responsible for intervening for any other signs or symptoms such as administering antihistamines or antipyretic medications.

Blunt trauma

To the abdomen may result from motor vehicle crashes, falls, blows, or explosions Commonly associated with extra abdominal injuries to the chest , head, or extremities * challenge because injuries may be difficult to detect*

Emergency management

Traditionally refers to care given to pts with urgent and critical needs Includes the concept that an emergency is whatever the pt or family considers it to be

Quality and safety nursing alert

** THE INFUSION RATE IS DETERMINED BY THE SEVERITY OF THE BLOOD LOSS AND THE CLINICAL EVIDENCE OF HYPOVOLEMIA. ANY BLOOD REPLACEMENT THERAPY SHOULD BE GIVEN VIA WARMER WHEN POSSIBLE, BECAUSE ADMINISTRATION OF LARGE AMTS OF BLOOD THAT HAS BEEN REFRIGERATED HAS A CORE COOLING EFFECT THAT MAY LEAD TO CARDIAC ARREST AND COAGULOPATHY**

Quality and safety nursing alert

** To decompress the bladder and monitor urine output in a pt with genitourinary injury, an indwelling catheter is inserted after a rectal examination has been completed, not before. In addition, urethral catheter insertion when a possible urethral injury is present is contraindicated; a urology consultation and further evaluation of the urethra are required

Quality and safety nursing alert

**IN CASE OF POTENTIAL FACIAL TRAUMA OR BASAL SKULL FRACTURE, THE NASOPHARYNGEAL AIRWAY SHOULD NOT BE USED BECAUSE IT COULD ENTER THE BRAIN CAVITY INSTEAD OF THE PHARYNX ***

Collection of forensic evidence

-When clothing is removed from the patient who has experienced trauma, the nurse must be careful not to cut through or disrupt any tears, holes, blood stains, or dirt present on the clothing if criminal activity is suspected. -Each piece of clothing should be placed in an individual paper bag. -If the clothing is wet, it should be hung to dry. -Clothing should not be given to families. -Valuables should be inventoried and either placed in the hospital safe or it should be clearly documented to which family member they were given. -If a police officer is present to collect clothing or any other items from the patient, each item is labeled. -The transfer of custody to the officer, the officer's name, the date, and the time are documented. All deaths of trauma pts should be reported to the medical examiner All lines and tubes should remain place Cover the pt's hands with paper bag to protect evidence

A nurse is caring for a patient with multiple injuries and performs the following. Place these actions in the order in which the nurse would perform them. Use all options. 1 Establish airway and ventilation 2 Prevent and treat shock 3 Control hemorrhage 4 Assess for head and neck injuries 5 Assess for abdomen, back, and extremity injuries 6 Splint fractures

1 Establish airway and ventilation 2 Control hemorrhage 3 Prevent and treat shock 4 Assess for head and neck injuries 5 Assess for abdomen, back, and extremity injuries 6 Splint fractures When providing care to a patient with multiple injuries, the nurse would first establish airway and ventilation, then control hemorrhage, prevent and treat hypovolemic shock, and assess for head and neck injuries. Then the nurse would evaluate for other injuries including re-assessing the head, neck, and chest and assessing the abdomen, back, and extremities. Then the nurse would splint fractures and, lastly, perform a more thorough and ongoing examination and assessment.

Debriefing typically occurs

1 to 10 days after the critical incident Participating staff are encouraged to discuss their feelings about the incident and are reassured that their negative feelings are normal and that they will diminish over time Participants should have a feeling of closure and be able to resume their professional roles at an emotional level commensurate to that of prior to the critical incident at the end of these sessions

A finger sweep is only to be used in which client population? A. Adolescent B. Conscious adult C. Unconscious adult D. Child

C. Unconscious adult A finger sweep should be used only in the unconscious adult client. This action draws the tongue away from the back of the throat and away from any foreign body that may be lodged there. A finger sweep should not be done on a conscious adult, child, or adolescent.

A nurse is performing a primary survey on a patient. Place the following steps in the order in which the nurse would perform them. 4 Use the head-tilt, chin-lift manuever 1 Provide ventilations 2 Assess pulses 3 Evaluate neurologic function

4 Use the head-tilt, chin-lift manuever 1 Provide ventilations 2 Assess pulses 3 Evaluate neurologic function A primary survey follows the ABCD sequence: airway, breathing, circulation, and disability. First, the nurse establishes a patent airway by repositioning the head, using the head-tilt, chin-lift or jaw-thrust maneuver, or inserting an airway. Next, the nurse provides adequate ventilations and then evaluates and restores cardiac output, which includes assessing pulses. Lastly, the nurse determines neurologic disability by assessing neurologic function using the Glasgow Coma Scale.

A quick neurologic assessment may be performed using the AVPU mnemonic

A - alert ( is pt alert and responsive) V- verbal ( does the pt respond to verbal stimuli? P - pain. Does the pt respond only to painful stimuli U - unresponsive. Is the pt unresponsive to all stimuli, including pain

ABCDE approach developed for injury prevention

A - describes assessment of the community for common injury mechanisms B - is used to describe building a coalition of key community members C- refers to communicating awareness of the trauma mechanisms and risks prevalent in the local community D - stands for developing and implementing interventions, which may be educational or legislative E- refers to evaluating the injury prevention program soon after it is launched

Hemorrhage that results in the reduction of circulating blood volume is

A main cause of shock

A client arrives at the emergency department and is experiencing a severe allergic reacton to a bee sting. The client received treatment and is being discharged. Which client statement indicates that additional teaching about exposure prevention is needed? A. "Brightly colored clothes help to ward off bees." B. "I should always wear something on my feet when I'm outside." C. "If a bee comes near me, I should stay still." D. "I need to avoid using perfumes and scented soaps when I'm going outside."

A. "Brightly colored clothes help to ward off bees." To prevent insect stings, the client should avoid wearing brightly colored clothing because it attracts bees. The client should wear covering on the feet and avoid going barefoot because yellow jackets nest and pollinate on the ground. Staying still or motionless reduces the likelihood of being stung. Perfumes and scented soaps attract bees and should be avoided.

The nurse is providing care for a client who is experiencing alcohol withdrawal. The client reports, "I cannot fall or stay asleep." The nurse observes that the client is agitated, having difficulty falling asleep and crying uncontrollably, with confused speech and a tachycardic pulse. Which intervention should the nurse implement first? A. Administer lorazepam as ordered by the health care provider B. Ask a family member to remain with the client C. Assess the client for suicidal and homicidal ideation D. Encourage the client to use deep breathing

A. Administer lorazepam as ordered by the health care provider Clients with alcohol withdrawal syndrome show signs of anxiety, uncontrollable fear, tremor, irritability, agitation, insomnia, and incontinence. They are talkative and preoccupied and experience visual, tactile, olfactory, and auditory hallucinations that often are terrifying. Autonomic overactivity occurs and is evidenced by tachycardia, dilated pupils, and profuse perspiration. Usually, all vital signs are elevated in the alcoholic toxic state. The goals of management are to give adequate sedation and support to allow the client to rest and recover without danger of injury or peripheral vascular collapse. A sufficient dosage of a benzodiazepine medication such as lorazepam should be administered to establish and maintain sedation, which reduces agitation, prevents exhaustion, prevents seizures, and promotes sleep. Although the alternate answer options should be included in the client's care, the nurse's first action should be to treat the presenting symptoms. Once the client is calm, the nurse can assess for the risk to harm self or others. The nurse can also support the client in managing anxiety by encouraging deep breathing. If a family member is present, the nurse can ask him or her to stay at the bedside to support the client and ensure safety as the client experiences withdrawal symptoms.

Nursing students are reviewing information about endotracheal intubation. They demonstrate understanding of the information when they identify which of the following as a reason for this procedure? Select all that apply. A. Allow connection to a manual resuscitation bag B. Decrease tracheobronchial secretions C. Facilitate removal of an upper airway obstruction D. Prevent aspiration into the lungs E. Establish an airway for ventilation

A. Allow connection to a manual resuscitation bag D. Prevent aspiration into the lungs E. Establish an airway for ventilation Endotracheal intubation is indicated to establish an airway for a patient who cannot be adequately ventilated with an oropharyngeal airway, bypass an upper airway obstruction, prevent aspiration, permit connection to a resuscitation bag or mechanical ventilator, or facilitate removal of tracheobronchial secretions.

Which of the following would the nurse identify as indicating that a client is experiencing a complete airway obstruction? Select all that apply. A. Clutching of the neck B. Cyanosis C. Spontaneous coughing D. Stridor E. Inability to speak

A. Clutching of the neck B. Cyanosis D. Stridor E. Inability to speak Manifestations of a complete airway obstruction include the inability to speak, breathe, or cough; clutching the neck; inspiratory and expiratory stridor; and cyanosis (a late sign). If the client can cough spontaneously, then a partial airway obstruction is most likely.

The ED staff work collaboratively and follow the ABCDE method to establish and treat health priorities effectively in a client experiencing a trauma. Which action is completed by the nurse when implementing the "D" element of this method? A. Assessing the client's Glasgow Coma Scale score B. Providing cervical spine protection C. Managing hypothermia D. Undressing the client quickly

A. Assessing the client's Glasgow Coma Scale score The primary survey focuses on stabilizing life-threatening conditions. The ED staff work collaboratively and follow the ABCDE (airway, breathing, circulation, disability, exposure) method. While implementing the D element, the nurse determines neurologic disability by assessing neurologic function using the Glasgow Coma Scale and performing a motor and sensory evaluation of the spine. A quick neurologic assessment may be performed using the AVPU mnemonic: A, alert: is the client alert and responsive? V, verbal: does the client respond to verbal stimuli? P, pain: does the client respond only to painful stimuli? U, unresponsive: is the client unresponsive to all stimuli, including pain? The other interventions are not included in this element of the primary survey.

The nurse is caring for a client who is agitated and confused. The client is persistently trying to get out of bed and attempted to remove the peripheral IV. The nurse has attempted to re-orient the client; however, this was not effective in de-escalating the client's agitation. The client yells, "I am going to punch you in the face!" What is the nurse's next action? A. Call security personnel to assist B. Apply physical restraints C. Move out of the client's view D. Adminster antipsychotic medication

A. Call security personnel to assist Clients at risk for harming staff members require specific interventions. It is important to first notification of security and administration of the potential for violence. Although medication and physical restraints maybe required, the nurse will not be able to carry out these interventions in a safe manner independently. The nurse should first call for security personnel to assist, all other interventions can be carried out with the support of trained staff. When a client is agitated and has the potential to be violent, they should not be left unattended. Moving out of the client's view can lead to further agitation for the client and increase the risk for escalating to violence.

When assessing a client with suspected carbon monoxide poisoning, which finding would be least reliable? A. Cherry red skin color B. Palpitations C. Confusion D. Headache

A. Cherry red skin color Skin color can range from pink or cherry-red to cyanotic and pale is not a reliable sign. In clients with carbon monoxide poisoning, central nervous system signs such as headache and confusion predominate. Palpitations also may occur.

A patient with a history of major depressive disorder is brought to the emergency department by a friend, who reports that the patient took an overdose of prescribed amitriptyline. Which of the following findings would the nurse expect to assess? Select all that apply. A. Clonus B. Hypoactive reflexes C. Tachycardia D. Visual hallucinations E. Hypothermia

A. Clonus C. Tachycardia D. Visual hallucinations Amitriptyline is a tricyclic antidepressant. In cases of overdose, the patient would likely experience tachycardia, hypotension, confusion, visual hallucinations, clonus, tremors, hyperactive reflexes, seizures, blurred vision, flushing, and hyperthermia.

A client undergoes a total abdominal hysterectomy. When assessing the client 10 hours later, the nurse identifies which finding as an early sign of shock? A. Confusion B. Heart rate of 110 beats/minute C. Pale, warm, dry skin D. Urine output of 30 ml/hour

A. Confusion Early in shock, inadequate perfusion leads to anaerobic metabolism, which causes metabolic acidosis. As the respiratory rate increases to compensate, the client's carbon dioxide level decreases, causing alkalosis and subsequent confusion and combativeness. Inadequate tissue perfusion causes pale, cool, clammy skin (not pale, warm, dry skin). An above-normal heart rate is a late sign of shock. A urine output of 30 ml/hour is within normal limits.

A client presents to the ED following a motor vehicle collision. The client is suspected of having internal hemorrhage. The nurse assesses the client for signs and symptoms of shock. Which are signs and symptoms of shock? Select all that apply. A. Cool, moist skin B. Increasing urine volume C. Delayed capillary refill D. Decreasing blood pressure E. Increasing heart rate

A. Cool, moist skin C. Delayed capillary refill D. Decreasing blood pressure E. Increasing heart rate Signs and symptoms of shock include cool, moist skin (resulting from poor peripheral perfusion), decreasing blood pressure, increasing heart rate, delayed capillary refill, and decreasing urine volume.

A high school football player is brought to the emergency department after collapsing at practice in extremely hot and humid weather. Which of the following would lead the nurse to suspect that the client is experiencing heat stroke? A. Delirium B. Temperature of 101 degrees F (38 degrees C) C. Bradycardia D. Diaphoresis

A. Delirium Manifestations of heat stroke include a temperature of 105 degrees F or greater (40.5 degrees C or greater), anhidrosis (absence of sweating), central nervous system dysfunction (bizarre behavior, delirium, confusion, or coma), hot, dry skin, tachycardia, tachypnea, and hypotension.

A patient is brought to the ED by a friend, who states that a tree fell on the patient's leg and crushed it while they were cutting firewood. What priority actions should the nurse perform? (Select all that apply.) A. Elevating the site to limit the accumulation of fluid in the interstitial spaces B. Applying a clean dressing to protect the wound C. Performing a fasciotomy D. Inserting an indwelling catheter E. Splinting the wound in a position of rest to prevent motion

A. Elevating the site to limit the accumulation of fluid in the interstitial spaces B. Applying a clean dressing to protect the wound E. Splinting the wound in a position of rest to prevent motion Major soft tissue injuries are dressed and splinted promptly to control bleeding and pain. If an extremity is injured, it is elevated to relieve swelling and pressure.

Which triage category refers to life-threatening or potentially life-threatening injury or illness requiring immediate treatment? A. Emergent B. Immediate C. Nonacute D. Urgent

A. Emergent The client triaged as emergent must be seen immediately. The triage category of urgent refers to minor illness or injury needing first-aid-level treatment. The triage category of immediate refers to nonacute, non-life-threatening injury or illness.

A patient brought to the ED by the rescue squad after getting off a plane at the airport is complaining of severe joint pain, numbness, and an inability to move the arms. The patient was on a diving vacation and went for a last dive this morning before flying home. What is a priority action by the nurse? A. Ensure a patent airway and that the patient is receiving 100% oxygen. B. Send the patient to the hyperbaric chamber. C. Draw labs for a chemistry panel. D. Send the patient for a chest x-ray.

A. Ensure a patent airway and that the patient is receiving 100% oxygen. Decompression sickness, also known as "the bends," occurs in patients who have engaged in diving (lake/ocean diving), high-altitude flying, or flying in commercial aircraft within 24 hours after diving. Signs and symptoms include joint or extremity pain, numbness, hypesthesia, and loss of range of motion. A patent airway and adequate ventilation are established before all other interventions, as described previously, and 100% oxygen is administered throughout treatment and transport.

Medical and nursing interventions for patients who present with multiple injuries follow a sequence of treatment priorities. Which of the following is the first priority of care? A. Establish an airway. B. Control hemorrhage. C. Assess for head and neck injuries. D. Prevent hypovolemic shock.

A. Establish an airway. The immediate intervention is to always manage the airway and breathing first; controlling hemorrhage is the second priority, followed by preventing and treating hypovolemic shock.

In which triage category would the nurse include a client who requires simple first aid or basic primary care? A. Fast track B. Urgent C. Emergent D. Nonurgent

A. Fast track Fast track patients require simple first aid or basic primary care and may be treated in the ED or safely referred to a clinic or physician's office. Urgent patients have serious health problems that are not immediately life threatening. They must be seen within 1 hour. Emergent patients have the highest priority, their conditions are life threatening and they must be seen immediately. Nonurgent patients have episodic illness that can be addressed within 24 hours without increased morbidity.

A female patient was sexually assaulted when leaving work. When assisting with the physical examination, what nursing interventions should be provided? (Select all that apply.) A. Label all torn or bloody clothes and place each item in a separate brown bag so that any evidence can be given to the police. B. Have the patient shower or wash the perineal area before the examination. C. Assess and document any bruises and lacerations. D. Ensure that the police are present when the examination is performed. E. Record a history of the event, using the patient's own words.

A. Label all torn or bloody clothes and place each item in a separate brown bag so that any evidence can be given to the police. C. Assess and document any bruises and lacerations. E. Record a history of the event, using the patient's own words. A history is obtained only if the patient has not already talked to a police officer, social worker, or crisis intervention worker. The patient should not be asked to repeat the history. Any history of the event that is obtained should be recorded in the patient's own words. The patient is asked whether he or she has bathed, douched, brushed his or her teeth, changed clothes, urinated, or defecated since the attack, because these actions may alter interpretation of subsequent findings. Each item of clothing is placed in a separate paper bag. The bags are labeled and given to appropriate law enforcement authorities. The patient is examined (from head to toe) for injuries, especially injuries to the head, neck, breasts, thighs, back, and buttocks. The exam focuses on external evidence of trauma (bruises, contusions, lacerations, stab wounds).

Which solution should the nurse use to replace lost fluids in a client with signs and symptoms of shock due to hemorrhaging? A. Lactated Ringer solution B. Type O negative blood C. Dextrose 5% in water D. Hypertonic saline

A. Lactated Ringer solution Replacement fluids may include isotonic electrolyte solutions( lactated Ringers, nomoral saline) ,colloids, and blood component therapy. O negative blood is prepared for emergency use in women of childbearing age. Dextrose 5% in water should not be used to replace fluids in hypovolemic clients. Hypertonic saline is used only to treat severe symptomatic hyponatremia and should be used only in intensive care units.

Which solid organ is most frequently injured in a penetrating trauma? A. Liver B. Pancreas C. Lung D. Brain

A. Liver The most frequently injured solid organ in a penetrating trauma is the liver because of its size and anterior placement in the right upper quadrant of the abdomen.

A client with a history of allergies comes to the emergency department. The nurse suspects anaphylaxis based on which of the following? Select all that apply. A. Localized itching B. Chest tightness C. Facial angioedema D. Pallor E. Increasing blood pressure

A. Localized itching B. Chest tightness C. Facial angioedema D. Pallor Manifestations suggesting anaphylaxis include chest tightness, generalized itching, pallor, massive facial angioedema, tachycardia or bradycardia, and decreasing blood pressure (as a result of peripheral vascular collapse).

The nurse is administering antivenin to a patient who was bitten on the arm by a poisonous snake. What intervention provided by the nurse is required prior to the procedure and every 15 minutes after? A. Measure the circumference of the arm. B. Assess peripheral pulses. C. Administer diphenhydramine (Benadryl). D. Administer cimetidine (Tagamet).

A. Measure the circumference of the arm. Before administering antivenin and every 15 minutes thereafter, the circumference of the affected part is measured. Premedication with diphenhydramine (Benadryl) or cimetidine (Tagamet) may be indicated, because these antihistamines may decrease the allergic response to antivenin. Antivenin is administered as an IV infusion whenever possible, although intramuscular administration can be used.

A nurse who is a member of an emergency response team anticipates that several patients with airway obstruction may need a cricothyroidotomy. For which of the following patients would this procedure be appropriate? Select all that apply. A. Patient with extensive facial trauma B. Patient with a lumbar spine injury C. Patient who is bleeding from the chest D. Patient with laryngeal edema secondary to anaphylaxis E. Patient with an obstructed larynx

A. Patient with extensive facial trauma D. Patient with laryngeal edema secondary to anaphylaxis E. Patient with an obstructed larynx Cricothyroidotomy is used in emergencies when endotracheal intubation is either not possible or contraindicated. Examples include airway obstruction from extensive maxillofacial trauma, cervical spine injury, laryngospasm, laryngeal edema after an allergic reaction or extubation, hemorrhage into neck tissue, and obstruction of the larynx.

When preparing to perform abdominal thrusts on a client with an airway obstruction, which of the following would be most appropriate? A. Positioning the hands in the midline slightly above the umbilicus B. Using a sequence of four thrusts, each progressing in intensity C. Placing the thumb side of one hand at the xiphoid process D. Having the conscious client lie down

A. Positioning the hands in the midline slightly above the umbilicus When performing abdominal thrusts, the nurse would place the thumb side of one fist against the client's abdomen in the midline slighlty above the umbilicus and well below the xiphoid process, grasping the fist with the other hand. Then the nurse would press the fist into the client's abdomen with a quick inward and upward thrust such that each new thrust should be a separate and distinct maneuver. The unconscious client is positioned on the back. The client who is conscious should be standing or sitting.

A nurse is providing care to the family of a client who was brought to the emergency department and suddenly died. Which of the following would be appropriate for the nurse to do? Select all that apply. A. Provide a private place for the family to be together. B. Provide sedation to family members as needed. C. Ask the family if they would like to view the body. D. Talk with the family about the client having "passed on." E. Allow the family to express their emotions freely.

A. Provide a private place for the family to be together. C. Ask the family if they would like to view the body. E. Allow the family to express their emotions freely. When providing care to a family experiencing the sudden death of a member, the nurse would take the relatives to a private place where they can be together to grieve. In addition, the nurse would encourage the family to view the body if they wish and allow members to support each other and express their emotions freely. Euphemisms such as "passing on" or "going to a better place" should be avoided. Sedation is avoided because it may mask or delay the grieving process, which is necessary to achieve emotional equilibrium and prevent prolonged depression.

A nurse is performing a primary survey of a client brought to the emergency department. Which of the following would the nurse include? Select all that apply. A. Providing adequate ventilation B. Assessing neurologic function C. Establishing airway patency D. Applying monitoring devices E. Obtaining a complete health history

A. Providing adequate ventilation B. Assessing neurologic function C. Establishing airway patency The primary survey addresses airway, breathing, circulation, and disability. The nurse would establish a patent airway, provide adequate ventilation, evaluate and restore cardiac output, and determine neurologic disability by assessing neurologic function. Obtaining a complete health history and applying monitoring devices are activities involved with the secondary survey.

A client is brought to the emergency department with suspected genitourinary injury. The nurse prepares the client for insertion of an indwelling urinary catheter for bladder decompression and urine output monitoring. The nurse reviews the client's medical record to ensure that which of the following has been completed? A. Rectal examination B. Diagnostic peritoneal lavage C. Computed tomography scan D. Bladder ultrasound

A. Rectal examination In a client with a suspected genitourinary injury, an indwelling urinary catheter is inserted for bladder decompression and urine output monitoring only after a rectal examination has been completed. Computed tomography or bladder ultrasound are not necessary. A diagnostic peritoneal lavage is a backup procedure for evaluating intraperitoneal injury.

A nurse is establishing a patient's airway. Which action would the nurse perform first? A. Repositioning the patient's head B. Giving abdominal thrusts C. Inserting an artificial airway D. Using the jaw-thrust maneuver

A. Repositioning the patient's head Establishing an airway may be as simple as repositioning the patient's head to prevent the tongue from obstructing the pharynx. Subsequent measures would include abdominal thrusts to dislodge a foreign body, head-tilt chin-lift or jaw-thrust manuever, or insertion of an artificial airway.

The nurse has commenced a transfusion of fresh frozen plasma (FFP) and notes the client is exhibiting symptoms of a transfusion reaction. After the nurse stops the transfusion, what is the next required action? A. Run a normal saline line to keep the vein open B. Remove the peripheral IV line C. Obtain a blood culture from the IV insertion site D. Start a dextrose 5% water infusion

A. Run a normal saline line to keep the vein open If the nurse suspects a transfusion reaction, the transfusion must be stopped immediately and the nurse's next action is to ensure the normal saline line is running at a rate that permits administration of IV fluids or medications that are required to treat the reaction. The nurse should ensure IV access is maintained. The 'to keep vein open (TKVO) rate allows the nurse to keep the IV client without the potential to cause fluid volume overload. It would be unsafe for the nurse to remove the peripheral IV because continued access is required for urgent IV administration of medications or fluids to treat the reaction. Obtaining a blood culture at the IV site would be necessary if an infection was suspected. This is not required for a transfusion reaction. Normal saline is the solution of choice when transfusing blood products because there is a risk for incompatibility with all other IV solutions.

Nursing students are reviewing information about anaphylactic reactions and their possible causes. The students demonstrate understanding of this information when they identify which of the following as a common cause? Select all that apply. A. Shellfish B. Medications C. Latex D. Insect stings E. Green vegetables F. Eggs

A. Shellfish B. Medications C. Latex D. Insect stings F. Eggs Common causes of anaphylactic reactions include insect stings, medications (eg, penicillin, iodinated-contrast materials), latex, insect stings, eggs, peanuts, and shellfish. Green vegetables typically are not associated with anaphylaxis.

A client comes to the emergency department after experiencing a wound. Inspection reveals an opening in the skin with distinct edges and whose depth is greater than the length of the wound. The nurse documents this as which type of wound? A. Stab B. Patterned C. Laceration D. Avulsion

A. Stab A stab wound is an incision of the skin with well-defined edges and is typically deeper than long. It is usually caused by a sharp instrument. A laceration is a tear in the skin with irregular edges and vein bridging. An avulsion is manifested as a tearing away of tissue from the supporting structures. A patterned wound takes on the outline of the object causing the wound.

A nurse is providing inservice education for staff members about evidence collection after sexual assault. The educational session is successful when staff members focus their initial care on which step? A. Supporting the client's emotional status B. Obtaining consent for examination C. Performing the pelvic examination D. Collecting semen

A. Supporting the client's emotional status The teaching session is successful when staff members focus first on supporting the client's emotional status. Next, staff members should gain consent to perform the pelvic examination, perform the examination, and collect evidence, such as semen if present.

When providing care to a client who has experienced multiple trauma, which of the following would be most important for the nurse to keep in mind? A. The client is assumed to have a spinal cord injury until proven otherwise. B. Injuries have occurred to at least three distinct organ systems. C. Most multiple trauma victims exhibit evidence of the trauma. D. The most lethal injuries are often the most readily apparent.

A. The client is assumed to have a spinal cord injury until proven otherwise. With clients experiencing multiple trauma, the nurse must assume that the client has a spinal cord injury until proven otherwise. Multiple trauma cleints experience life-threatening injuries to at least two distinct organs or organ systems. Evidence of the trauma may be sparse or absent. Additionally, the injury that may seem the least significant may be the most lethal.

The nurse is caring for a client in the intensive care unit who is recovering from trauma as a result of a motor vehicle accident that claimed the life of the client's friend. While the nurse is performing a dressing change on the client's surgical wound, the client states, "I don't deserve to live. I have just been thinking about ending it all." As the nurse assesses the client's imminent risk for suicide, what contributing factors need to be considered? Select all that apply. A. The client's maternal uncle committed suicide. B. The client attempted suicide as a teenager. C. The client is not able to ambulate unassisted. D. The client's parents visit on a daily basis. E. The client had a close relationship to the accident victim.

A. The client's maternal uncle committed suicide. B. The client attempted suicide as a teenager. E. The client had a close relationship to the accident victim. When assessing a client's suicide risk, it is very important to first determine whether the client has a previous history of suicide attempts. Having a suicide-attempt history increases the risk that the client will attempt to end his or her life if experiencing suicidal thoughts. Having a family member who has committed suicide increases the risk that the client will follow through with a suicide attempt. Family support mitigates the risk that the client will follow through with a suicide attempt if the client is experiencing hopeless thoughts. Having a close relationship with the victim in the car accident indicates the client is experiencing grief and loss and may increase the risk of suicide. If the client is unable to ambulate unassisted, this decreases the client's means to access to be able to follow through with a suicide attempt.

A nurse is preparing to assist with a gastric lavage for a client who has ingested an unknown poison and is obtunded. To ensure that the tube reaches the stomach, the nurse would measure the distance from the bridge of the nose to which of the following? A. Xiphoid process B. Chin C. Umbilicus D. Ear lobe

A. Xiphoid process The nurse measures the tube from the bridge of the nose to the xiphoid process to ensure that the tube reaches the stomach on insertion.

A client is admitted to the ED after a near-drowning accident. The client is diagnosed with saltwater aspiration. The nurse will observe the client for several hours to monitor for symptoms of A. pulmonary edema. B. head injury. C. hypothermia. D. hyponatremia.

A. pulmonary edema. Resultant pathophysiologic changes and pulmonary injury depend on the type of fluid (fresh or salt water) and the volume aspirated. Freshwater aspiration results in a loss of surfactant and therefore an inability to expand the lungs. Saltwater aspiration leads to pulmonary edema from the osmotic effects of the salt within the lungs. If a person survives submersion, acute respiratory distress syndrome, resulting in hypoxia, hypercarbia, and respiratory or metabolic acidosis, can occur. The client would experience hypernatremia. Hypothermia and head injury may be associated with near drowning but would be apparent at the time of admission and would not develop after several hours.

Most common reasons for ED visits in 2013 were

Abdominal pain Chest pain Cough Fever

Rewarming

Active internal (core) rewarming and passive (spontaneous) or active external rewarming Active warming - used for moderate to severe hypothermia less than 28C to 32.2C (82.5F to 90F) and include cardiopulmonary bypass, warm fluid administration, warm humidified oxygen and warmed peritoneal lavage. Monitoring for ventricular fibrillation as the pt's temp increases from 31C to 32C (88F to 90F) is essential Passive or active external rewarming is used for mild hypothermia 32.2 to 35C (90F to 95F) - uses over the bed heaters to the extremities and increases blood flow to the acidotic, anaerobic extremities. The cold blood from the peripheral tissues has high lactic acid levels - as this blood returns to the core, causes a significant drop in the core temp ( core temp afterdrop) and can potentially cause cardiac dysrhythmias and electrolyte disturbances Forced air warming blankets - care to prevent burn due to decreased sensation

Management of fractures

Assessment of ABC which includes pulses in the extremities Evaluation for neurologic or abdominal injuries before the extremity is treated, unless a pulseless extremity is detected Repositioning of the extremity to proper alignment if a pulseless extremity is involved Pulseless fractured femur - Hare traction (portable inline traction device) If repositioning does not restore the pulse- a rapid total body assessment is completed and the pt is transferred to the operating room for arteriography and possible arterial repair vs amputation A splint is applied before the pt is moved

Emergency Severity Index (ESI)

Assigns pts into five levels from level 1 (most urgent)to level 5 (least urgent) Assigned to triage levels based on both their acuity and anticipated needs

Anxiety and denial

At this stage give honest answers at the level of the family's understanding Prolonged denial is not encouraged or supported Encourage asking questions Encourage the family members to recognize and talk about their feelings of anxiety

Which of the following statements would most lead a nurse to suspect that a patient is experiencing food poisoning? A. "I've been feeling sick to my stomach for about 3 or 4 days now." B. "My brother got sick like me after eating the same food." C. "I have a pain in my left side, down low near my groin." D. "The food I ate seemed to look and taste like it should."

B. "My brother got sick like me after eating the same food." The statement about the patient's brother also being sick after eating the same food suggests food poisoning. Feeling sick to the stomach for 3 to 4 days could indicate various problems, not just food poisoning. Food tasting or looking fine does not really indicate anything definitive about the patient's condition. Most foods causing bacterial poisoning do not have unusual odor or taste. A pain in the left groin area is more suggestive of appendicitis, not food poisoning.

The nurse is caring for an intensive care unit client who has died with family members at the bedside. The death was sudden and unexpected resulting from a car accident that took place three days ago. The family is upset and the client's partner, crying loudly, yells, "How did this happen? We were just about to celebrate his birthday. He can't be gone!" The family member continues to cry inconsolably. How should the nurse respond? A. " I will get you some medication that will help you feel more calm." B. "We did everything we could possibly do to try to save his life." C. "He has passed on to a better place now." D. "It is important to face the reality that he is gone."

B. "We did everything we could possibly do to try to save his life." In order to help the family cope with the sudden death of their loved one, it is helpful for the nurse to explain that the care team employed all medical interventions possible to try to save the client's life. With the support of other members of the health care team, the nurse can take the time to explain what life saving treatments were rendered. The nurse should avoid giving sedation to family members; this may mask or delay the grieving process, which is necessary to achieve emotional equilibrium and to prevent prolonged depression. It is important that the nurse avoid using euphemisms such as "passed on." Instead the nurse should show the family that he or she cares by touching, and offering coffee, water, and the services of a chaplain. The nurse should encourage the family to express emotion including events leading up to the event that led to the client's death. The nurse should not challenge initial feelings of anger or denial.

Permanent brain injury or death will occur within which time frame secondary to hypoxia? A. 1 to 2 minutes B. 3 to 5 minutes C. 6 to 8 minutes D. 9 to 10 minutes

B. 3 to 5 minutes If the airway is completely obstructed, permanent brain injury or death will occur within 3 to 5 minutes secondary to hypoxia. Air movement is absent in the presence of complete airway obstruction. Oxygen saturation of the blood decreases rapidly because obstruction of the airway prevents air from entering the lungs. Oxygen deficit occurs in the brain, resulting in unconsciousness, with death following rapidly. The other time frames are incorrect.

The nurse is admitting a patient with a penetrating abdominal injury from a knife wound. What should the nursing measures for a penetrating abdominal injury include? (Select all that apply.) A. Covering any protruding viscera with sterile dressings soaked in normal saline solution B. Assessing for manifestations of hemorrhage C. Exploring the abdominal wound with a gloved finger D. Looking for any associated chest injuries E. Irrigating the wound with normal saline and a syringe

B. Assessing for manifestations of hemorrhage A. Covering any protruding viscera with sterile dressings soaked in normal saline solution D.Looking for any associated chest injuries Hemorrhage frequently accompanies abdominal injury, especially if the liver or spleen has been traumatized. Therefore, the patient is assessed continuously for signs and symptoms of external and internal bleeding. If abdominal viscera protrude, the area is covered with sterile, moist saline dressings to keep the viscera from drying.

A homeless client presents to the ED. Upon assessment, the client is experiencing hypothermia. The nurse will plan to complete which priority intervention during the rewarming process? A. Administer inotropic drugs B. Attach a cardiac monitor C. Insert a Foley urinary catheter D. Assist with endotracheal intubation

B. Attach a cardiac monitor Continuous electrocardiograph (ECG) monitoring is performed during the rewarming process because cold-induced myocardial irritability leads to conduction disturbances, especially ventricular fibrillation. A urinary catheter should be inserted to monitor urinary output; however, ECG monitoring is the priority. There is no indication for endotracheal intubation. Inotropic medications are contraindicated because they can stimulate the heart and increase the risk for fatal dysrhythmias, such as ventricular fibrillation.

The nurse is caring for a client who has just been intubated and started on mechanical ventilation in the intensive care unit. The nurse recognizes that it is possible to inadvertently intubate the right lung only. What nursing assessment and monitoring is required to determine if this complication has occured? Select all that apply. A. Re-set the ventilator rate as needed B. Auscultate both sides of the chest C. Mark the endotracheal tube at the corner of the mouth and nose D. Apply suctioning to clear the airway E. Monitor for both high and low pressure alarms

B. Auscultate both sides of the chest C. Mark the endotracheal tube at the corner of the mouth and nose E. Monitor for both high and low pressure alarms It is important to remember that the right main bronchus is wider, shorter, and more vertical than the left. This physiologic difference may lead to inadvertent intubation of the right lung only. It is essential to listen to both sides of the chest for bilateral breath sounds, mark the correct endotracheal tube (ETT) placement at lip or nares, and monitor for high- and low-pressure alarms. Although suctioning the airway to remove secretions is an essential part of the nurse's responsibility when caring for a ventilated client, this action will not help the nurse determine if the tube has been placed only into the right lung only. The ventilator settings are determined by the client's primary health provider and any changes would require an order. These settings are specific to the client's individualized needs. Despite this, the re-seting the ventilator would not help determine incorrect placement of the endotracheal tube.

A patient working in a chemical facility sustains a chemical burn to his arms. The chemical involved was white phosphorus. Which of the following would be the priority nursing action? A. Covering the burned area to prevent further spread B. Brushing off all traces of the chemical from the patient's skin C. Applying ice to the burned area D. Dousing the area with large amounts of water

B. Brushing off all traces of the chemical from the patient's skin For a chemical burn involving lye or white phosphorous, all evidence of the chemical should be brushed off the patient before any flushing occurs. These chemicals, if exposed to water, have the potential for exploding or for deepening the burn. Covering the burn area or applying ice is an inappropriate action.

A client is brought to the emergency department with severe hemorrhage requiring masssive blood replacement. The nurse warms the blood in a commercial warmer based on the understanding that infusion of large amounts of blood could result in which of the following? A. Hemolytic transfusion reaction B. Cardiac arrest C. Hyperthermia D. Fluid overload

B. Cardiac arrest Blood must be warmed in a commercial blood warmer because administration of large amounts of blood that has been refigerated has a core cooling effect that may lead to cardiac arrest and coagulopathy. Hyperthermia, hemolytic transfusion reaction, or fluid overload is not the concern.

The nurse has come on shift to find that a client newly admitted to the ICU is confused and persistently trying to get out of bed despite being comforted and re-oriented by the nurse. The client begins to pull on the peripheral intravenous line in the hand and speaking in non-sensical terms. The client's history indicates a sudden onset of neurological symptoms after developing a bacterial infection. The nurse anticipates providing care for which health problem? A. Fever B. Delirium C. Pain D. Anxiety

B. Delirium Delirium is a confused state that has a sudden onset and can last hours to days or weeks; it is characterized by hyperactivity and has the potential to be reversible. The client who quickly becomes confused and agitated while attempting to pull out IV lines and get out of bed is experiencing delirium. The nurse caring for this client should anticipate the need to provide close monitoring to prevent injury. Although clients can experience a high level of stress with both pain and anxiety, which often accompany one another, these problems do not cause confusion and disorientation. Nursing interventions would be aimed at reducing pain and anxiety with the use of medications and other non-pharmacological interventions that enhance client comfort. Although fever can accompany delirium, it does not produce confusion and disorientation on its own.

A nurse suspects an older adult is experiencing heat stroke based on which assessment findings? Select all that apply. A. Weakness B. Delirium C. Increased thirst D. Bradypnea E. Temperature 105 degrees F (40.6 degrees C) F. Lack of sweating

B. Delirium E. Temperature 105 degrees F (40.6 degrees C) F. Lack of sweating A patient with heat stroke typically exhibits a temperature of 105 degrees F (40.6 degrees C) or higher; profound central nervous system dysfunction; hot, dry skin; anhidrosis (absence of sweating); tachypnea; hypotension; and tachycardia. Increased thirst and weakness would suggest heat exhaustion.

The nurse is caring for a client in the intensive care unit and while reviewing the client's history, the nurse notes the client had a King laryngeal tube inserted to begin ventilation. The nurse recognizes this intervention was required for which reason? A. The client was hemorrhaging into the neck. B. Emergency response personnel performed this intervention outside the hospital. C. Laryngeal edema prevented placement of an endotracheal tube. D. The client's airway is oversized requiring a specialized endotracheal tube.

B. Emergency response personnel performed this intervention outside the hospital. If the client is not hospitalized and cannot be intubated in the field, emergency medical personnel may insert a King laryngeal tube, which rapidly provides pharyngeal ventilation. When the tube is inserted into the trachea, it functions like an endotracheal tube. An oversized airway would not be the rationale for the use of a King tube. This is a temporary life-saving intervention used by first responders. Once admitted to hospital and stabilized, a different type of endotracheal tube would be considered. Laryngeal edema and hemorrhage into the neck are two emergency conditions in which intubation may not be an option. In this case, the nurse would note in the client's history that an emergency cricothyroidotomy (cricothyroid membrane puncture) was performed to establish an airway.

A client is being cared for in the ED. The client is assigned to the triage category of "urgent." How often must the nurse reassess the client? A. Every 15 minutes B. Every 30 minutes C. Every 60 minutes D. Every 120 minutes

B. Every 30 minutes Clients assigned to the resuscitation category must receive continuous nursing surveillance, those in the emergent category must be reassessed at least every 15 minutes, clients in the urgent category must be reassessed at least every 30 minutes, those in the less urgent category must be reassessed at least every 60 minutes, and those in the nonurgent category must be reassessed at least every 120 minutes.

The nurse is providing care to a client who will be ambulating for the first time after being extubated. The client tells the nurse, "I don't want to do this today. It's too soon and I am afraid I am not strong enough." What intervention should the nurse implement first for the client's fear of falling? A. Evaluate the client for cognitive impairment B. Explore possible causes of the client's fear C. Clear the area around the bed D. Allow the client to remain on bedrest

B. Explore possible causes of the client's fear The client is exhibiting a fear of falling. For a client who has not mobilized in days due to mechanical ventilation and other medication interventions in the intensive care unit (ICU), ICU-acquired weakness is a reality. The client's concerns should be addressed by exploring the possible reasons for the fear of falling first. The client may be experiencing pain, dizziness or self-doubt. By identifying this cause, the nurse will be able to formulate the next action. The risk for falls is not due to cognitive impairment. This is evident in that the client is aware of current limitations and as a result is fearful. Preventative and rehabilitative measures to counter ICU-acquired weakness generally include early identification and treatment of potential causes of multiple organ failure (in particular severe sepsis and septic shock), avoiding unnecessary deep sedation and hyperglycemia, promotion of early mobilization, and thoughtful decisions regarding the risks versus benefits of corticosteroids. For these reasons, the client should not be encouraged to continue to have bedrest. Although the nurse should ensure the area around the bed is free of clutter to prevent a fall, this does not address the client's anxiety related to the fear of falling.

After inserting an oropharyngeal airway, which of the following indicates that the airway is properly positioned? A. Distal end is in the pharynx. B. Flange is at the client's lips. C. Tongue lies on top of the airway. D. Air is moving through the airway.

B. Flange is at the client's lips. An oropharyngeal airway is properly positioned when the distal end is in the hypopharynx and the flange is approximately at the client's lips. Air moving through the airway may or may not indicate proper placement. An oropharyngeal airway is inserted so that the tongue is displaced anteriorly.

The nurse is caring for a client with diabetes who requires a peripheral intravenous (PIV) line for antibiotic administration and to treat dehydration. The nurse must avoid inserting which type of PIV? A. Upper arm B. Foot C. Hand D. Forearm

B. Foot PIV lines should rarely be used in the foot for various reasons. They limit the client's ability to ambulate and tend to occlude easily. These types of IVs should never be used in clients with diabetes due to the risk that the client has neuropathy and cannot feel injury caused by the IV catheter. IV lines in the forearm and hands are acceptable and are commonly used sites. These sites would be safe to use for a client with diabetes. The upper arm is a site of choice for the insertion of a peripherally inserted central line (PICC) not a PIV line. Although, this site would not be an option for a PIV line, it would be safe for use in a client with diabetes if warranted.

Nursing students are reviewing the categories of intra-abdominal injuries. The students demonstrate understanding of the information when they identify which of the following as examples of penetrating trauma? Select all that apply. A. Fall from a roof B. Gunshot wound C. Being struck with a baseball bat D. Motor-vehicle crash E. Knife-stab wound

B. Gunshot wound E. Knife-stab wound Examples of penetrating trauma include gunshot wounds and stab wounds. Motor vehicle crashes, falls, and being struck with a baseball bat are examples of blunt trauma.

A client present to the ED following a work-related injury to the left hand. The client has an avulsion of the left ring finger. Which correctly describes an avulsion? A. Denuded skin B. Tissue tearing away from supporting structures C. Skin tear with irregular edges and vein bridging D. Incision of the skin with well-defined edges, usually long rather than deep

B. Tissue tearing away from supporting structures An avulsion is described as a tearing away of tissue from supporting structures. A laceration is a skin tear with irregular edges and vein bridging. Abrasion is denuded skin. A cut is an incision of the skin with well-defined edges, usually long rather than deep.

A nurse is providing care to a client who is a victim of trauma resulting from injuries sustained in a convenience store robbery. The client has been stabbed numerous times in the abdomen and chest. His shirt is bloody and torn. Which of the following would be most appropriate when collecting forensic evidence? A. Placing the client's clothing in a plastic bag B. Hanging up any damp or wet clothing to dry before securing C. Placing the law enforcement officer's name on the secured clothing for pick up D. Cutting away clothing through the tears or holes

B. Hanging up any damp or wet clothing to dry before securing When collecting forensic evidence, the nurse should remove the client's clothing, being careful not to cut through or disrupt any tears, holes, blood stains or dirt present on the clothing. Each piece of clothing is put into a separate paper bag and labeled. If the clothing is wet or damp, it should be hung to dry. If a police officer is present to collect clothing or any other items from the client, each bag is labeled with the client's name, and the transfer of custody to the officer, the officer's name, date, and time are documented.

A client comes to the emergency department with a suspected airway obstruction. The emergency department team prepares to manage the client as if he has a complete airway obstruction based on which of the following? A. Wheezing between coughs B. High-pitched noise on inhalation C. Refusal to lie flat D. Forceful coughing

B. High-pitched noise on inhalation A client who demonstrates a weak, ineffective cough, high-pitched noise while inhaling, increased respiratory difficulty, or cyanosis should be managed as if he or she has a complete airway obstruction. Forceful coughing, wheezing between coughs, and a refusal to lie flat suggest a partial airway obstruction that can be managed as such.

A nurse is caring for a client who has arrived at the emergency department in shock. The nurse intervenes based on the knowledge that which of the following is the most common cause of shock? A. Sepsis B. Hypovolemia C. Anaphylaxis D. Cardiac dysfunction

B. Hypovolemia Types of shock include cardiogenic, neurogenic, anaphylactic, and septic. Of these, the most common cause is hypovolemia.

A client suffering from carbon monoxide poisoning would exhibit which manifestation? A. Cherry red skin coloring B. Intoxication C. Hyperactivity D. Severe hypertension

B. Intoxication A client suffering from carbon monoxide poisoning appears intoxicated (from cerebral hypoxia). Other signs and symptoms include headache, muscular weakness, palpitation, dizziness, and mental confusion. The skin coloring in the client with carbon monoxide poisoning can range from pink to cherry red to cyanotic and pale and is not a reliable diagnostic sign.

A nurse is completing her annual cardiopulmonary resuscitation training. The class instructor tells her that a client has fallen off a ladder and is lying on his back; he is unconscious and isn't breathing. What maneuver should the nurse use to open his airway? A. Head tilt-chin lift B. Jaw-thrust C. Seldinger D. Abdominal thrust

B. Jaw-thrust If a neck or spine injury is suspected, the jaw-thrust maneuver should be used to open the client's airway. To perform this maneuver, the nurse should position herself at the client's head and rest her thumbs on his lower jaw, near the corners of his mouth. She should then grasp the angles of his lower jaw with her fingers and lift the jaw forward. The head tilt-chin lift maneuver is used to open the airway when a neck or spine injury isn't suspected. To perform this maneuver the nurse places two fingers on the chin and lifts while pushing down on the forehead with the other hand. The abdominal thrust is used to relieve severe or complete airway obstruction caused by a foreign body. The Seldinger maneuver is a method of percutaneous introduction of a catheter into a vessel.

A nurse is providing care to an older adult client who has frostbite of the feet. Which action would be least appropriate? A. Providing an analgesic for pain B. Massaging the feet C. Restricting ambulation D. Placing sterile cotton between the toes after rewarming

B. Massaging the feet For a client with frostbite, massaging the affected body part is contraindicated. Analgesia is given for pain during the rewarming process because it can be very painful. Ambulation would be restricted. Once rewarmed, sterile gauze or cotton is placed between the affected toes to prevent maceration.

A client presents to the ED after an unsuccessful suicide attempt. The client is diagnosed with an acetaminophen overdose. The nurse anticipates the administration of which medication? A. Diazepam B. N-acetylcysteine C. Flumazenil D. Naloxone

B. N-acetylcysteine Treatment of acetaminophen overdose includes administration of N-acetylcysteine. Flumazenil is administered in the treatment of nonbarbiturate sedative overdoses. Naloxone is administered in the treatment of narcotic overdoses. Diazepam may be administered to treat uncontrolled hyperactivity in the client with a hallucinogen overdose.

The nurse is caring for a patient in the ED who is breathing but unconscious. In order to avoid an upper airway obstruction, the nurse is inserting an oropharyngeal airway. How would the nurse insert the airway? A. With the concave portion touching the posterior pharynx B. Upside down and then rotated 180 degrees C. At an angle of 90 degrees D.With the convex portion facing upward

B. Upside down and then rotated 180 degrees The nurse should insert the oropharyngeal airway with the tip facing up toward the roof of the mouth until it passes the uvula and then rotate the tip 180 degrees so that the tip is pointed down toward the pharynx. This displaces the tongue anteriorly, and the patient then breathes through and around the airway.

The nurse is caring for a client with known myocardial ischemia. The client will be getting up to ambulate for the first time in three days after being on bedrest since admission to the intensive care unit. Which medication should the nurse administer before the client ambulates? A. Vasopressin B. Nitroglycerin C. Norepinephrine D. Dobutamine

B. Nitroglycerin Nitroglycerin is indicated for use in clients who experience angina pectoris as a result of myocardial ischemia. The medication acts by decreasing blood pressure and causing arterial vasodilation permitting blood flow into the myocardium. Nitroglycerin should be given prior to any anticipated physical exertion that is likely to bring on chest pain from vasoconstriction. Vasopressin is a vasoactive medication administered to increase blood pressure in cases where clients have diabetes insipidus, a gastrointestinal bleed or in cases of septic shock. Norepinephrine raises blood pressure and is indicated for use in emergencies such as cardiac arrest or for hypovolemia. Dobutamine increased heart contractility and blood pressure to improve stroke volume in clients with congestive heart failure.

A client presents to the ED following a chemical burn. The client identifies the source of the burn as white phosphorus. The nurse knows that treatment will include A. alternately applying water and ice to the burn. B. No application of water to the burn. C. immediately drenching the skin with running water from a shower, hose, or faucet. D. washing off the chemical using warm water, then flushing the skin with cool water.

B. No application of water to the burn. Water should not be applied to burns from lye or white phosphorus because of the potential for an explosion or for deepening of the burn. All evidence of these chemicals should be brushed off the client before any flushing occurs.

A patient is brought to the emergency department following an overdose of a selective serotonin reuptake inhibitor (SSRI). While assessing the patient, the nurse suspects that the patient may be developing serotonin syndrome based on which of the following? A. Lack of perspiration B. Seizures C. Hypotension D. Lethargy

B. Seizures Serotonin syndrome is manifested by agitation, seizures, hyperthermia, diaphoresis, and hypertension.

The intensive care unit nurse is assessing a client who is going to require a peripheral intravenous (PIV) line for fluids. The nurse should consider what information in the client's health history when deciding the site for the PIV? A. The client has a fluid volume restriction B. The client has had a mastectomy on the right side C. The client has a history of falls D. The client has hypertension

B. The client has had a mastectomy on the right side Contraindications to the placement of a PIV line in any specific placement (right vs. left side) will include history of mastectomy, arterial-venous shunt placement, peripherally inserted central catheter (PICC) line placement, thrombus, trauma, and other device placements, such as splints and casts. The nurse will only have the option to start the PIV on a site in the client's left arm if the client has had a ride-sided mastectomy. A history of hypertension does not preclude the client from having a PIV inserted in any specific location. Although fluid requirements are monitored more strictly with clients who are on a fluid volume restriction, this does not influence the placement of the PIV. The nurse should always be aware of the risks of a PIV for a client with a falls history. The tubing can be a tripping hazard, therefore, the client with a falls history who requires a PIV should be closely monitored but this does not preclude the client from having a PIV inserted.

An emergency nurse has collected evidence from a patient who was shot during a robbery. The nurse is preparing to transfer the evidence to law enforcement. Which of the following would be important for the nurse to include when documenting this transfer? Select all that apply. A. The labels placed on the collection bags B. Time of the transfer of evidence C Family members who witnessed the transfer D. Name of the law-enforcement official E. Date that the evidence was collected

B. Time of the transfer of evidence D. Name of the law-enforcement official E. Date that the evidence was collected When transferring evidence to law enforcement, the nurse must document the chain of custody. This includes the information that evidence was transferred to the officer, the officer's name, and the date and time of the transfer. Labels are placed on each item, but this does not need to be documented for the transfer. The names of family members witnessing the transfer also do not need to be documented.

Which data is bimportant for the nurse to record while assessing the client with an open wound? A. vital signs B. Time when the client last received a tetanus immunization

B. Time when the client last received a tetanus immunization If the client has an open wound, the nurse ascertains when the client last received a tetanus immunization. This vital information helps assess the risk of infection in a client with an open wound. The assessment begins with measuring the client's vital signs. It is important to ascertain the time and place of injury with the degree of movement and range of motion in all cases, not just in the case of an open wound.

Following an earthquake, a client who was rescued from a collapsed building is seen in the emergency department. He has blunt trauma to the thorax and abdomen. The nursing observation that most suggests the client is bleeding is: A. diminished breath sounds. B. orthostatic hypotension. C. a recent history of warfarin (Coumadin) usage. D. a prolonged partial thromboplastin time (PTT).

B. orthostatic hypotension. Bleeding is a volume-loss problem, which causes a drop in blood pressure. As the bleeding persists and the body's ability to compensate declines, orthostatic hypotension becomes evident. A prolonged PTT and a history of warfarin usage are causes of bleeding but aren't evidence of bleeding. As bleeding persists and the client's level of consciousness declines, breathing will become more shallow and breath sounds will diminish; however, this is a late and unreliable manifestation of bleeding.

The antibacterial agent povidine-iodine (Betadine) should not

Be allowed to get deep into the wound without thorough rinsing It is only used for initial cleansing because it injures exposed and healthy tissue, resulting in further tissue damage

Discharge planning

Before discharge, verbal and written instructions for continuing care are given to the pt and the family or significant others Instructions should include information about prescribed meds, treatments, diet, activity, and when to contact a health care provider or schedule follow up appts

The nurse in the hospital emergency department is assessing a patient who fell while intoxicated with alcohol. The nurse is using the Clinical Institute Withdrawal Assessment-Alcohol (CIWA-A) scale to assess the patient's need for a benzodiazipine medication. In order to assess for auditory disturbances, which question should the nurse ask the patient? A. "Are you experiencing any burning or numbness?" B. "Are you finding the light is too harsh or bothering your eyes?" C. "Are you hearing anything that is disturbing you?" D. "Does it feel like there is a tight band around your head?"

C. "Are you hearing anything that is disturbing you?" The Clinical Institute Withdrawal Assessment-Alcohol (CIWA-A) scale is used in the assessment of alcohol withdrawal. The patient's score on this scale helps determine the level of intervention that is required to support safe, withdrawal from alcohol. Assessing for auditory disturbances is one subsection on the scale. In order to effectively assess for this symptom, the nurse should ask the patient if they are hearing anything that is disturbing. By asking the patient if they are experiencing any numbness or burning would help to assess for tactile disturbances. By asking the patient if the light is bothering their eyes would support the assessment for visual disturbances. Asking the patient if it feels like there is a tight band around their head would help determine if the patient has a headache or fullness of the head. These are all symptom items that are measured by this scale.

The nurse is caring for a client in the ED following a sexual assault. The client is hysterical and crying. The client states, "I know I'm pregnant now, maybe I have HIV. Why did this happen to me?" Which is the best response by the nurse? A. "Would you like us to complete HIV testing?" B. "Do you want the phone number for the National Sexual Assault Hotline?" C. "Let's talk about this. Do you want me to call a support person?" D. "Do you want to discuss antipregnancy measures?"

C. "Let's talk about this. Do you want me to call a support person?" The client should be reassured that anxiety is natural and asked whether a support person may be called. The goals of management are to provide support, reduce the client's emotional trauma, and gather available evidence for possible legal proceedings. Throughout the client's stay in the ED, the client's privacy and sensitivity must be respected. The client may exhibit a wide range of emotional reactions, such as hysteria, stoicism, or feelings of being overwhelmed. Support and caring are crucial.

A patient is brought to the emergency department. Assessment reveals that the patient is lethargic and diaphoretic and complaining of right upper quandrant pain. Acetaminophen toxicity is suspected and an acetaminophen level is drawn. Which result would the nurse interpret as indicating toxicity for the patient if he weighs 70 kg? A. 9100 mg B. 6300 mg C. 10,500 mg D. 7700 mg

C. 10,500 mg An acetaminophen level greater than or equal to 140 mg/kg would be considered toxic. For a patient weighing 70 kg, the toxic level would be 9800 mg. A level of 10,500 mg would be greater, thus indicating toxicity.

A client suspected of acetaminophen (Tylenol) toxicity reports that he ingested the medication at 7 p.m. At what time should the nurse anticipate laboratory tests to assess the acetaminophen level? A. Stat B. 24 hours from the last dose C. 11:00 p.m. D. 8 p.m.

C. 11:00 p.m. The duration of action of acetaminophen ranges from 3 to 5 hours. Its half-life ranges from 1 to 3 hours. At least 4 hours should pass between the last dose and laboratory assessment of the acetaminophen level.

A patient with frostbite to both lower extremities from exposure to the elements is preparing to have rewarming of the extremities. What intervention should the nurse provide prior to the procedure? A. Massage the extremities. B. Elevate the legs. C. Administer an analgesic as ordered. D. Apply a heat lamp.

C. Administer an analgesic as ordered. During rewarming, an analgesic for pain is administered as prescribed, because the rewarming process may be very painful. To avoid further mechanical injury, the body part is not handled. Massage is contraindicated.

A 40-year-old client is admitted to the ED with facial bruises and a broken right wrist. Upon further assessment, the nurse notes multiple bruises in various stages of healing. Which is the best course of action by the nurse? A. Asking the client how the various bruises were obtained B. Contacting the local police and reporting the suspected abuse C. Asking the client whether they are experiencing abuse D. Providing the client with information about local shelters

C. Asking the client whether they are experiencing abuse The priority is to ask the client whether they are experiencing abuse, then proceed as the situation dictates. Nurses must be mindful that competent adults are free to accept or refuse the help that is offered to them. Some clients insist on remaining in the home environment where the abuse or neglect is occurring. The wishes of clients who are competent and not cognitively impaired should be respected. However, all possible alternatives, available resources, and safety plans should be explored with the client. Mandatory reporting laws in most states require health care workers to report suspected child abuse or abuse of older adults to an official agency, usually Adult (or Child) Protective Services. All that is required for reporting is the suspicion of abuse; the health care worker is not required to prove abuse or neglect.

A client is brought to the emergency department after being involved in a motor vehicle collision. Which of the following would lead the nurse to suspect internal bleeding? A. Bradycardia B. Rising blood pressure C. Delayed capillary refill D. Pale pink dry skin

C. Delayed capillary refill If a client exhibits tachycardia, falling blood pressure, thirst, apprehension, cool moist skin, or delayed capillary refill, internal bleeding should be suspected.

A nurse is preparing an in-service education program about emergency nursing to a group of newly hired nurses who will be working in the emergency department. When describing the current status of visits to the emergency department, which of the following would the nurse include in the presentation? A. The majority of clients arriving at the emergency department arrive by ambulance. B. Clients, on average wait about a hour before being seen by a health care provider. C. Clients with Medicaid use the emergency department more often than clients with private health insurance. D. Heart attacks and stroke account for most of the visits to the emergency department.

C. Clients with Medicaid use the emergency department more often than clients with private health insurance. According to the most recent survery, clients with Medicaid use emergency departments more often than clients with private health insurance, Medicare, or self-pay. More than 15.5% of clients arrived at the emergency department by ambulance, leaving the majority of clients arriving by other means. Injuries account for almost one-half of all emergency department visits. The average emergency department waiting time before being seen by a health care provider for definitive treatment is approximately 2.4 hours.

Which term refers to injuries that occur when a person is caught between objects, run over by a moving vehicle, or compressed by machinery? A. Blunt trauma B. Penetrating abdominal injuries C. Crush injuries D. Intra-abdominal injuries

C. Crush injuries Crush injuries are those that occur when a person is caught between objects, run over by a moving vehicle, or compressed by machinery. Blunt trauma is commonly associated with extra-abdominal injuries to the chest, head, or extremities. Penetrating abdominal injuries include those such as gunshot wounds and stab wounds. Intra-abdominal injuries are categorized as penetrating and blunt trauma.

The health care team in an intensive care unit have experienced a critical incident in which a young client died unexpectedly and the client's father physically attacked the senior physician treating the client. The client's father was arrested and escorted from the intensive care unit by police, against his will and in handcuffs. A critical incident stress management (CISM) staff meeting held 3 days after the incident took place. What would be the purpose for that meeting? A. Defusing B. Follow up C. Debriefing D. Counselling

C. Debriefing After serious events, critical incident stress management (CISM) is necessary to critique individual and group performance and to facilitate healthy coping. Optimally, this may consist of three steps: defusing, debriefing, and follow-up. Debriefing typically occurs 1 to 10 days after the critical incident. Debriefing sessions follow a format similar to the initial defusing session; however, during these sessions, participating staff are encouraged to discuss their feelings about the incident and are reassured that their negative reactions and feelings are normal and that their negative feelings will diminish over time. Defusing occurs immediately after the critical incident. During this session, affected staff are encouraged to discuss their feelings about the incident and are given contact information so that they may talk to someone if they have disturbing symptoms (e.g., sleeplessness, excessive worry). Follow-up may occur after the debriefing session is completed for those participants who have persistent negative symptoms and may consist of continued individual or group counseling and therapy. Counseling or group therapy would typically occur outside the context of the stress-inducing environment. Individuals may require private counseling versus group counseling.

The nurse is conducting a secondary survey on a client in the ED. Which action is completed during the secondary survey? A. Establishing a patent airway B. Assessment of peripheral pulses C. Diagnostic and laboratory testing D. Undressing the client

C. Diagnostic and laboratory testing Diagnostic and laboratory testing is completed during the secondary survey, along with a complete health history, a head-to-toe assessment, insertion or application of monitoring devices, splinting of suspected fractures, cleansing, closure, and dressing of wounds, and performance of other necessary interventions based on the client's condition. The other interventions are completed during the primary survey.

A patient is brought to the emergency department and diagnosed with decompression sickness. The nurse interprets this as indicating that the patient most likely has been involved with which of the following? A. Swimming in a lake B. Working in a chemical plant C. Diving in an ocean D. Running a race in hot humid weather

C. Diving in an ocean Decompression sickness occurs when patients have engaged in diving in a lake or ocean or high-altitude flying or flying in a commercial aircraft within 24 hours of diving. Swimming in a lake could lead to a near-drowing episode. Running a race in hot humid weather would increase a person's risk for heat stroke. Working in a chemical plant would increase the risk for chemical burns.

A client is brought to the emergency department by ambulance. The client is seriously ill and unconscious. No family or friends are present. Which of the following would be most appropriate to do? A. Ask the ambulance team for information about the client's family to ensure informed consent. B. Explain to the client that care is going to be provided because he is seriously ill. C. Document the client's condition and absence of friends or family for obtaining consent to treatment. D. Check the client's record for the name of a family member to call to allow care to be provided.

C. Document the client's condition and absence of friends or family for obtaining consent to treatment. Consent is needed to examine and treat a client unless he or she is unconscious or in critical condition and unable to make decisions. In this situation, the client is unconscious and no friends or family are around to provide consent to treatment. The nurse should document this fact and provide care. Checking the client's record and asking the ambulance team for information would waste valuable time. Explaining to the client that care will be provided is appropriate even though the client is unconscious, but documentation is essential.

A patient arrives at the emergency department after taking more than 20 lorazepam tablets. Which of the following would the nurse anticipate that the patient would be given to reverse the effects of the drug? A. N-aceytlcysteine B. Naloxone C. Flumazenil D. Diazepam

C. Flumazenil Lorazepam is a nonbarbiturate sedative whose effects are reversed with flumazenil. Naloxone is used to reverse the effects of opioids. Diazepam is used to treat seizures associated with drug overdose. It would not be used here, because it is in the same class as lorazepam and concurrent administration would add to the patient's overdose state. N-acetylcysteine is the antidote for acetaminiophen toxicity.

A nurse is working as a camp nurse during the summer. A camp counselor comes to the clinic after receiving a snakebite on the arm. What is the first action by the nurse? A. Apply a tourniquet to the arm above the bite. B. Make an incision and suck the venom out. C. Have the patient lie down and place the arm below the level of the heart. D. Apply ice to the area.

C. Have the patient lie down and place the arm below the level of the heart. Initial first aid at the site of the snakebite includes having the person lie down, removing constrictive items such as rings, providing warmth, cleansing the wound, covering the wound with a light sterile dressing, and immobilizing the injured body part below the level of the heart. Airway, breathing, and circulation are the priorities of care. Ice, incision and suction, or a tourniquet is not applied.

The nurse educator is providing orientation to a group of nurses newly hired to an intensive care unit. The group of nurses are correct in stating which is the most common type of shock managed in critical care? A. Anaphylactic B. Neurogenic C. Hypovolemic D. Cardiogenic

C. Hypovolemic The underlying cause of shock (hypovolemic, cardiogenic, neurogenic, anaphylactic, or septic) must be determined. Of these, hypovolemia is the most common cause.

The nurse is caring for a client suffering from carbon monoxide poisoning. The nurse will expect the client to exhibit which manifestation? A. Severe hypertension B. Cherry red skin coloring C. Intoxication D. Hyperactivity

C. Intoxication A client suffering from carbon monoxide poisoning appears intoxicated (from cerebral hypoxia). Other signs and symptoms include headache, muscular weakness, palpitation, dizziness, and mental confusion. The skin coloring in the client with carbon monoxide poisoning can range from pink to cherry red to cyanotic and pale and is not a reliable diagnostic sign.

A patient arrives at the emergency department after sustaining a gunshot wound to the abdomen. When assessing the patient, the nurse pays particular attention to which of the following? A. Stomach B. Large intestine C. Liver D. Kidneys

C. Liver Penetrating abdominal injuries, such as from a gunshot wound, are serious and result in a high incidence of injury to hollow and solid organs. Although any organs can be injured, the liver is the most frequently injured solid organ. The small bowel is a frequently injured hollow organ. Thus, of the options shown, the nurse would assess the liver area most closely.

A family member brings a client to the ED following an apparent oxycodone overdose. The client is experiencing severe respiratory depression. Which medication will the nurse administer? A. N-acetylcysteine B. Flumazenil C. Naloxone hydrochloride D. Diazepam

C. Naloxone hydrochloride Naloxone hydrochloride, a narcotic antagonist, reverses respiratory depression and coma. Flumazenil is a benzodiazepine antagonist. Diazepam is a benzodiazepine. N-acetylcysteine is used for acetaminophen toxicity.

A nurse is providing an in-service program for fellow emergency nurses about hypothermia and rewarming methods used. The nurse determines that the presentation was successful when the group identifies which of the following as a passive active rewarming method? A. Cardiopulmonary bypass B. Warmed humidified oxygen by ventilator C. Over-the-bed heaters D. Forced warm air blankets

C. Over-the-bed heaters Passive active rewarming uses over-the-bed heaters to the extremities and increases blood flow to the acidotic, anaerobic extremities. Cardiopulmonary bypass and warm humidified oxygen by ventilator are examples of active core (internal) rewarming methods. Forced warm air blankets are examples of active external rewarming methods.

A patient with intra-abdominal injuries is brought to the emergency department. Which of the following would most likely alert the nurse to suspect internal bleeding secondary to a ruptured spleen? A. Abdominal distention B. Rebound abdominal tenderness C. Pain in the left shoulder D. Contusion of the right upper quadrant

C. Pain in the left shoulder Pain in the left shoulder is common in a patient with bleeding from a ruptured spleen. Pain in the right shoulder is consistent with a laceration of the liver. The spleen is located in the left upper quadrant, not the right. Rebound tenderness and abdominal distention are generalized signs suggesting intraperitoneal injury. Although these generalized signs may accompany a ruptured spleen, they are less specific than pain in the left shoulder.

A nurse is providing care to a client in the emergency department and walks into the hallway to get equipment. All of a sudden, gunshots are heard. Which of the following would be the nurse's priority? A. Gaining control of the situation B. Securing the area C. Protecting himself or herself D. Providing care to the injured

C. Protecting himself or herself If gunfire occurs in the emergency department, self-protection is the priority. Security officers and police must gain control of the situation first and then care is provided to the injured.

The nurse in an intensive care unit is caring for a client who requires blood work to assess for changes in blood coagulation due to heparin therapy. Which test should the nurse expect to see prescribed for this value to be assessed? A. Arterial blood gas (ABG) B. Lactate C. Prothrombin time (PTT) D. White blood cell (WBC)

C. Prothrombin time (PTT) Prothrombin time (PTT) is assessed in the blood work to identify coagulopathy or presence of chemically induced anticoagulation. This client is receiving heparin, an intravenous medication that helps to prevent the formation of clots; therefore, the PTT must be monitored regularly to ensure the medication remains within the therapeutic range. The client's white blood cell (WBC) reflects a count of this blood component to detect elevation of these cells, which is related to increased physiological stress. Typically this stress is infection, but it can also increase when there is trauma. Lactate would be drawn with the blood work to determine acidosis and need for continued resuscitation. Arterial blood gas (ABG) is evaluated to determine pH for the presence of acidosis, the base deficit for resuscitation evaluation, and ventilation parameters (PaCO2, PaO2).

The nurse has received a client into care who was admitted with a heroin overdose. The client has a 5-year history of illicit substance use with cocaine, heroine and oxycodone. The client develops a sudden onset of wheezing, restlessness and a cough that produces a frothy, pink sputum. The nurse suspects the client has most likely developed which complication of opioid overdose? A. Pneumonia B. Panic attack C. Pulmonary edema D. Congestive heart failure

C. Pulmonary edema The nurse should suspect the client has developed pulmonary edema, which is frequently seen in clients who abuse/overdose on narcotics. Many drugs — ranging from illegal drugs such as heroin and cocaine to aspirin — are known to cause noncardiogenic pulmonary edema. Pneumonia is not the likely cause given the sudden onset of respiratory symptoms accompanied but coughing up the pink frothy sputum. The client's history of illicit substance use and now overdose on these drugs should lead the nurse to suspect pulmonary edema is the cause of the sudden onset of these symptoms over congestive heart failure, in which clients have a more gradual onset of respiratory issues. Although a panic attack can manifest in shortness or breath and restlessness, the client would not be wheezing or producing blood tinged sputum with a cough. Panic attacks do, however, have a sudden onset and can cause the client chest pain and a sense of doom.

The nurse in an intensive care unit is assigned to two clients. One of the clients has just passed away. The deceased client's family members have arrived to be at the client's bedside. Despite wanting to support the client's family, the nurse is must assess the other client's vital signs every 15 minutes, because the client is receiving a blood transfusion. In this situation, what is the nurse's best action? A. Explain to the family it is a busy time on the unit but someone will be with them soon B. Hand off care of the other client to another nurse C. Request that the pastor be present to support the family at the client bedside D. Delegate the blood transfusion to the licensed practical/vocational nurse

C. Request that the pastor be present to support the family at the client bedside The death of a family member in the intensive care unit is a difficult and often time-consuming process. If nurses are unable to spend much time with grieving client's family, it is imperative to find the family alternate help: a colleague with more experience with grieving clients, a pastor, a social worker, hospital volunteers, family, or friends. It would be best if the nurse requests a pastor be available to the family in advance of their arrival to the deceased client's bedside. Much of the pastor's role in hospital settings is to support grieving families; therefore, the pastor would have more time to be with the family during this difficult time. The blood transfusion in the intensive care unit is not within the scope of practice for the licensed practical/vocational nurse. The nurse cannot delegate the monitoring of blood products to this health care provider. The intensive care unit is a busy environment and as difficult as it is for the assigned nurse to remain with the deceased client, it would be even more difficult for a nurse with a full assignment to take on the support role for the family. Explaining to the family that the unit is busy demonstrates a lack of empathy and would be countertherapeutic communication. It would not be appropriate to explain this to the family.

A client is admitted to the emergency department after sustaining a penetrating injury to the abdomen. Which of the following would the nurse identify as a possible cause? A. Concrete debris from an explosion B. Impact of a steering wheel C. Stabbing with a knife D. Fall to the ground from a ladder

C. Stabbing with a knife Penetrating abdominal injuries are ones involving an opening into the abdomen, such as those that occur with a gunshot or stabbing. Blunt injuries usually occur with motor vehicle crashes, falls, and explosions.

A nurse is caring for a client who is experiencing alcohol withdrawal. Which statement best indicates that the client understands the need for long-term treatment? A. The client agrees to attend supportive counseling. B. The client agrees to ongoing participation in one or more support groups. C. The client agrees to detoxification, rehabilitation, and participation in an aftercare program. D. The client agrees to involve his family in psychotherapy.

C. The client agrees to detoxification, rehabilitation, and participation in an aftercare program. Detoxification, rehabilitation, and participation in an aftercare program are the only options that address the client's long-term treatment needs. Supportive counseling, family involvement, and support-group participation are important aspects of the treatment process, but they don't address the client's need for long-term treatment.

The nurse is caring for a client with right ventricular heart failure. The nurse understands hypervolemia will have what effect on the client's heart? A. The client's ventricles will not have to work as hard B. The client's will experience systemic vasodilation C. The client's myocardial oxygen requirements will be higher D. The client's stroke volume will be decreased

C. The client's myocardial oxygen requirements will be higher Clients with heart failure are typically hypervolemic and as a result this increases the cardiac preload. An increased fluid volume increases the stroke volume, ventricular work and myocardial oxygen requirements. Vasodilation can be a potential cause for decreased preload and afterload, not increased preload as in this case. This client would experience vasoconstriction due to the increase volume with each stroke.

A client is in hypovolemic shock. To determine the effectiveness of fluid replacement therapy, the nurse should monitor the client's: A. hemoglobin level. B. temperature. C. blood pressure. D. heart rate.

C. blood pressure. With adequate fluid replacement, fluid volume in the intravascular space expands, raising the client's blood pressure. The hemoglobin level reflects red blood cell concentration, not overall fluid status. Temperature and heart rate aren't directly related to fluid status.

Monitoring hypothermia

CABs of basic life support Vital signs Central venous pressure Urine output Arterial blood gas levels BUN Glucose Electrolytes Creatinine Chest X-rays Continuous ECG - cold induced myocardial irritability leads to conduction disturbances, esp ventricular fibrillation Arterial line to record bp and to facilitate blood sampling Core temp monitoring

Pts who hemorrhage are at risk for

Cardiac arrest caused by hypovolemia caused secondary to anoxia

Animal and human bites

Cat bites- risk of infection (Pasteurella in their saliva High of infection - human bites Photographs for evidence Cleanse bite wound with soap and water - administration of ABT and tetanus toxoid as prescribed

Manifestations of hypothermia

Changes in all organ systems Progressive deterioration, with apathy, poor judgement, ataxia, dysarthria, drowsiness, pulmonary edema, acid base abnormalities, coagulopathy, and eventual coma Shivering may be suppressed at a temp of less than 32.2C (90F), because the body's self warming mechanisms become ineffective Weak pulse and bp leads to undetectable peripheral pulses Hypoxemia Acidosis

Which phase of the psychological reaction to rape is characterized by fear and flashbacks? A. Reorganization phase B. Acute disorganization phase C. Denial phase D. Heightened anxiety phase

D. Heightened anxiety phase During the heightened anxiety phase, the client demonstrates anxiety, hyperalertness, and psychosomatic reactions, in addition to fear and flashbacks. The acute disorganization phase is characterized by shock, disbelief, guilt, humiliation, and anger. The denial phase is characterized by an unwillingness to talk. The reorganization phase occurs when the incident is put into perspective. Some clients never fully recover from rape trauma.

A pneumothorax (both simple and tension) or sucking (open) chest wound is managed with a

Chest tube and occlusion of the sucking wound; immediate relief of increasing positive intrathoracic pressure and maintenance of adequate ventilation should occur

Anger

Common in crisis situations as a way of handling anxiety and fear Directed to the pt but also to the physician, the nurse, or admitting personnel Allow the anger to be expressed and assist the family members to identify their feelings of frustration

Secondary survey includes the following

Complete health history, including history of current events Head to toe assessment (includes reassessment of airway and breathing parameters and vital signs) Diagnostic and laboratory testing Insertion or application of monitoring devices such as ECG electrodes, arterial lines, or urinary catheters Splinting of suspected fractures Cleansing, closure, and dressing of wounds Performance of other necessary interventions based on the pt's condition

A physical assessment is conducted with rewarming to observe for

Concomitant injury, such as soft tissue injury, dehydration, alcohol intoxication, or fat embolism Hyperkalemia from the release of potassium in the damaged cells and hypovolemia are corrected Risk of infection Tetanus prophylaxis and aseptic technique is used during dressing changes NSAIDs are given for pain

Canadian Triage and Acuity Scale (CTAS)

Consists of five levels which include time parameters that guide how frequently pts must be assessed by either a nurse or a provider

The goals of emergency management of hemorrhage are to

Control the bleeding Maintain adequate circulating blood volume for tissue oxygenation Prevent shock

Signs and symptoms of shock

Cool, moist skin (resulting from poor peripheral perfusion) Decreasing bp Tachycardia Delayed capillary refill Decreased urine output

The nurse is administering 100% oxygen to a patient with carbon monoxide poisoning and obtains a carboxyhemoglobin level. Which level would the nurse interpret as indicating that oxygen therapy can be discontinued? A. 6% B. 9% C. 7% D. 4%

D. 4% Oxygen is administered until the carboxyhemoglobin level is less than 5%.

The nurse is caring for a client in the ED with frostbite to the left hand. During the rewarming process of the hand, the nurse should perform which action? A. Rupture any hemorrhagic blebs that are noted. B. Keep the hand in the circulating bath for 1 hour. C. Have the client complete active range-of-motion exercises. D. Administer analgesic medications as ordered.

D. Administer analgesic medications as ordered. During rewarming, an analgesic for pain is administered as prescribed because the rewarming process may be very painful. Frozen extremities are usually placed in a 37°C to 40°C (98.6°F to 104°F) circulating bath for 30- to 40-minute spans. This treatment is repeated until circulation is effectively restored. Hemorrhagic blebs, which may develop 1 hour to a few days after rewarming, are left intact and unruptured. Nonhemorrhagic blisters are debrided to decrease the inflammatory mediators found in the blister fluid. After rewarming, hourly active motion of any affected digits is encouraged to promote maximal restoration of function and to prevent contractures.

The nurse received a patient from a motor vehicle accident who is hemorrhaging from a femoral wound. What is the initial nursing action for the control of the hemorrhage? A. Elevate the injured part. B. Apply a tourniquet. C. Immobilize the area to control blood loss. D. Apply firm pressure over the involved area or artery.

D. Apply firm pressure over the involved area or artery. Direct, firm pressure is applied over the bleeding area or the involved artery at a site that is proximal to the wound (Fig. 72-3). Most bleeding can be stopped or at least controlled by application of direct pressure. Otherwise, unchecked arterial bleeding results in death. A firm pressure dressing is applied, and the injured part is elevated to stop venous and capillary bleeding, if possible. If the injured area is an extremity, the extremity is immobilized to control blood loss. A tourniquet is applied to an extremity only as a last resort when the external hemorrhage cannot be controlled in any other way and immediate surgery is not feasible.

As part of an emergency department team, an emergency nurse is conducting a secondary survey on a client. Which of the following would the nurse include? A. Establishing a patent airway B. Providing adequate ventilation C. Assessing neurologic function D. Applying electrocardiogram electrodes

D. Applying electrocardiogram electrodes A secondary survey is completed after the primary survey priorities of airway, breathing, circulation, and disability have been addressed. Applying electrocardiogram electrodes would be a component of the secondary survey. Establishing a patent airway, providing adequate ventilation, and determining neurologic disability by assessing neurologic function are components of the primary survey.

A client with depression and behavioral changes is transferred from a local assisted living center to the emergency department. The nurse notes that the client cries out when she approaches. When the nurse gains the client's confidence and performs an assessment, the nurse notes bruising of the labia and a lateral laceration in the perineal area. When the nurse asks the client about the injury, the client shakes her head and begins to cry "don't tell, don't tell." The nurse suspects sexual abuse. How should the nurse proceed? A. Notify the rape crisis team. B. Notify the physician of her findings immediately. C. Notify the client's family. D. Attend to the client's physiological needs.

D. Attend to the client's physiological needs. The nurse should attend to the client's immediate physiological needs, including physical safety. Next, the nurse can notify the physician and the rape crisis team. The family should be notified if the client consents, but not until the rape investigation is complete.

A client has a gaping wound on his forearm that is bleeding profusely. Applying pressure to which pressure point would be most helpful? A. Subclavian B. Femoral C. Radial D. Brachial

D. Brachial The pressure point at the brachial artery would be most appropriate because this site is proximal to the bleeding site. The femoral pressure point would be useful for bleeding in the lower extremities. The radial pressure point would be appropriate for bleeding in the wrist and hands. The subclavian pressure point would be used for bleeding in the upper anterior chest area.

A patient who has accidentally ingested toilet bowel cleaner is brought to the emergency department. Which action would NOT be appropriate for the nurse to implement? A. Dilution with water or milk B. Gastric lavage C. Administration of activated charcoal D. Induced vomiting

D. Induced vomiting Vomiting is never induced after ingestion of caustic substances (acid or alkaline) such as toilet bowl cleaner because the substance is corrosive to the tissues. Appropriate actions include dilution with milk or water, gastric lavage, and administration of activated charcoal.

A client presents to the ED reporting choking on a chicken bone. The client is breathing spontaneously. The nurse applies oxygen and suspects a partial airway obstruction. Which action should the nurse do next? A. Prepare the client for a bronchoscopy. B. Insert a nasopharyngeal airway. C. Insert an oropharyngeal airway. D. Encourage the client to cough forcefully.

D. Encourage the client to cough forcefully. If the client can breathe and cough spontaneously, a partial obstruction should be suspected. The client is encouraged to cough forcefully and to persist with spontaneous coughing and breathing efforts as long as good air exchange exists. There may be some wheezing between coughs. If the client demonstrates a weak, ineffective cough, a high-pitched noise while inhaling, increased respiratory difficulty, or cyanosis, the client should be managed as if there were complete airway obstruction. If the client is unconscious, inspection of the oropharynx may reveal the offending object. X-ray study, laryngoscopy, or bronchoscopy also may be performed. There is no indication that an artificial airway is indicated.

The nurse is providing care to a client who was brought to hospital with a opioid overdose. The nurse should expect to include which immediate interventions in the care of this client? Select all that apply. A. Apply warming blankets to client B. Assess the client using the CIWA-A scale C. Assess respiratory rate every 4 to 6 hours D. Ensure the head of the bed remains elevated E. Monitor naloxone intravenous infusion

D. Ensure the head of the bed remains elevated E. Monitor naloxone intravenous infusion Interventions in the urgent care of a client who has overdosed on an opioid narcotic focuses on reversal of the effects of the narcotic agent and supporting oxygenation. The nurse should ensure the client has the head of the bed elevated to aid respirations and monitor the intravenous infusion of naloxone, an opioid narcotic reversal agent. Applying a warming blanket to a client in this state should not be considered an immediate intervention as the blanket may interfere with the nurse's ability watch respirations closely. This may also risk causing the client hyperthermia. The CIWA-A scale would be appropriate in assessing withdrawal from alcohol. Respirations need to be assessed more closely than every 4 to 6 hours when immediate, more urgent care is being provided to prevent respiratory depression

A patient has undergone a diagnostic peritoneal lavage. The nurse interprets which result as indicating a positive test? A. Red blood cell count of 50,000/mm3 B. Absence of bile C. White blood cell count of 300/mm3 D. Evidence of feces

D. Evidence of feces A diagnostic peritoneal lavage is considered positive if there is bile, feces, or food in the specimen, a red blood cell count greater than 100,000/mm3, and a white blood cell count greater than 500/mm3.

A patient is brought to the emergency department after being locked outside of her house in the frigid weather for several hours. The nurse suspects that the patient has sustained frostbite of her hand based on which of the following findings? A. Hand that is firm to palpation B. Hand that appears pink with some white spotting C. Hand that is cool with pale nailbeds D. Hand that is insensitive to touch

D. Hand that is insensitive to touch Indicators of frostbite include an extremity that is hard, cold, and insensitive to touch and appears white or mottled blue-white.

sustained in a snowboarding accident. The family has chosen to remove life support measures to allow the client's death. Upon hearing the family's decision, what is the nurse's first action? A. Ask the family if the client had advanced directives B. Assess for interrupted family processes C. Provide family members with PRN sedation D. Request senior medical staff discuss organ donation

D. Request senior medical staff discuss organ donation Clients who meet the criteria for past health and current diagnosis of brain death are eligible to donate organs to those on the various transplant lists. This places nurses in a difficult position at times due to their simultaneous obligations to care for a particular client and the family while informing organ donation services of a potential donor. When the diagnosis of brain death is made, it is usually up to the senior medical staff and organ procurement services to approach the family about the possibility of organ donation. The nurse's next best action is to request a senior physician speak to the family in a timely manner so organs can be harvested and made available as needed. Advance directives are typically in place for clients who are older and for whom death may be expected. In this case, the client is young and death is unexpected, advanced directives are not likely and this question would be inappropriate. Although the nurse should assess for interrupted family process, this is not the nurse's initial action after hearing the family has decided to remove life support. This nursing assessment goes beyond acute care and into the provision of community health services which the family will need throughout their grieving process.

Which guideline is appropriate for a nurse to implement while helping family members cope with the sudden death of a loved one? A. Provide details of the factors attendant to the sudden death. B. Obtain orders for sedation for family members. C. Inform the family that the client has passed on. D. Show acceptance of the body by touching it, giving the family permission to touch.

D. Show acceptance of the body by touching it, giving the family permission to touch. The nurse should encourage the family to view and touch the body if they wish, since this action helps the family to integrate the loss. The nurse should avoid using euphemisms such as "passed on." The nurse should avoid giving sedation to family members, because this may mask or delay the grieving process. The nurse should avoid volunteering unnecessary information (e.g., client was drinking at the time of the accident).

A patient was bitten by a tick 3 months ago and is now having muscle aches as well as joint pain and swelling. The patient is having difficulty with self care and requires assistance with activities of daily living (ADLs). What stage of Lyme disease does the nurse recognize the patient is in? A. Stage II B. Stage IV C. Stage I D.Stage III

D.Stage III Lyme disease has three stages. Stage I presents with a classic "bull's-eye" rash (i.e., erythema migrans) and flulike signs and symptoms that may include chills, fever, myalgia, fatigue, and headache. If antibiotics are not administered, stage II Lyme disease may present within 4 to 10 weeks following the tick bite and may manifest with joint pain, memory loss, poor motor coordination, and meningitis. Stage III can begin anywhere from weeks to more than a year after the bite and has serious long-term chronic sequelae, including arthritis, neuropathy, myalgia, and myocarditis.

An adolescent is brought to the ED after a vehicular accident and is pronounced dead on arrival (DOA). When the parents arrive at the hospital, what is the priority action by the nurse? A. Ask them to sit in the waiting room until she can spend time alone with them. B. Ask the emergency physician to medicate the parents so that they can handle their child's unexpected death quietly and without hysteria. C. Speak to one parent at a time in a private setting so that each can ventilate feelings of loss without upsetting the other. D. Speak to both parents together and encourage them to support each other and express their emotions freely.

D. Speak to both parents together and encourage them to support each other and express their emotions freely. The nurse should take the family to a private place and talk to the family together so that they can grieve together and hear the information given together. The nurse should encourage family members to support each other and to express emotions freely (grief, loss, anger, helplessness, tears, disbelief). The nurse should avoid giving sedation to family members; this may mask or delay the grieving process, which is necessary to achieve emotional equilibrium and to prevent prolonged depression.

A nurse is assessing a patient who is suspected of having a partial airway obstruction. Which of the following would the nurse expect to find? A. High-pitched noises on inhalation B. Cyanosis C. Severe respiratory distress D. Spontaneous coughing

D. Spontaneous coughing If a patient can breathe and cough spontaneously, a partial airway obstruction should be suspected. If the patient demonstrates a weak, ineffective cough, high-pitched noise while inhaling, increased respiratory difficulty, or cyanosis, the patient should be managed as if there were a complete airway obstruction.

When reviewing the results of a client's lumbar puncture, a nurse notes a glucose level of 32 mg/dl. What does this result suggest to the nurse? A. This glucose level is normal. B. The client may have undiagnosed diabetes mellitus because this level is elevated above the normal glucose level for cerebral spinal fluid (CSF). C. The client is experiencing hypoglycemia. D. The client may have bacterial meningitis.

D. The client may have bacterial meningitis. The normal glucose level for CSF ranges from 50 mg/dl to 75 mg/dl. The client's reduced glucose level may indicate a condition such as bacterial meningitis. The client's glucose level doesn't indicate diabetes mellitus. A decreased serum (not CSF) glucose level indicates hypoglycemia.

What is a common source of airway obstruction in an unconscious client? A. Edema B. Saliva or mucus C. A foreign object D. The tongue

D. The tongue In an unconscious client, the muscles controlling the tongue commonly relax, causing the tongue to obstruct the airway. When this situation occurs, the nurse should use the head-tilt, chin-lift maneuver to cause the tongue to fall back into place. If she suspects the client has a neck injury she must perform the jaw-thrust maneuver.

Control of external bleeding

Direct firm pressure is applied over the bleeding area or the artery involved at the site that is proximal to the wound Firm pressure dressing, and the injured part is elevated to stop venous and capillary bleeding Immobilization of the injured extremity A tourniquet is applied to an extremity when external hemorrhage can not be controlled any other way and until surgery can be performed - the tourniquet is placed proximal to the wound and tied tightly enough to control arterial blood flow

The nurse is assigned to a client admitted to the ICU from the emergency department. The client sustained multiple injuries from a motor vehicle accident. When reviewing the client chart, the notes indicate the client's emergency care was managed in what sequence of steps? 1Establish airway and start ventilation 2Examine client for additional injuries to the body 3Application of pressure to control abdominal bleeding 4Assess for head and neck injuries 5Reassess pulses and neurovascular status 6Start peripheral intravenous insertion and infusion of fluids

Establish airway and start ventilation Application of pressure to control abdominal bleeding Start peripheral intravenous insertion and infusion of fluids Assess for head and neck injuries Examine client for additional injuries to the body Reassess pulses and neurovascular status The goals of treatment are to determine the extent of injuries and to establish priorities of treatment. Any injury interfering with a vital physiologic function (e.g., airway, breathing, circulation) is an immediate threat to life and has the highest priority for immediate treatment. Essential lifesaving procedures are performed simultaneously by the emergency team. Establishing the airway and performing ventilation is necessary to support airway and breathing. Hypovolemic shock is prevented by applying pressure to bleeding sites and initiating a peripheral IV and immediate start of infusion of intravenous fluids. As soon as the client is resuscitated, clothes are removed or cut off and a rapid physical assessment is performed. The physical assessment should prioritize head and neck injuries and then injuries over the rest of the body. Ongoing examination, assessment and diagnostic evaluation are necessary. The health care team will continue to assess vascular and neurological status as these can change quickly.

Priority management in pts with multiple injuries

Establish airway and ventilation Control hemorrhage Prevent and treat hypovolemic shock Assess for head and neck injuries Evaluate for other injuries - expose and reassess head and neck, chest; assess abdomen, back and extremities Splint fractures and then reassess pulses and neurovascular status Perform a more thorough and ongoing examination and assessment; diagnostic studies

The primary survey focuses on stabilizing life threatening conditions via the ABCDE method

Establish patent airway Provide adequate ventilation, employing resuscitation measures when necessary, cervical spine protection to trauma pts and chest injuries assessed first, immediately after the airway is established Evaluate and restore cardiac output by controlling hemorrhage, preventing and treating shock, and maintaining or restoring effective circulation which includes prevention and management of hypothermia. Examination of peripheral pulses and any immediate closed reductions of fractures or dislocations are performed if an extremity is pulseless Determine neurologic disability by assessing neurologic function using the Glasgow Coma Scale (GCS) and a motor and sensory evaluation of the spine Undress the pt quickly but gently to so that any wounds or areas of injury are identified

Oropharyngeal airway

Is inserted over the back of the tongue into the lower posterior pharynx in a pt is breathing spontaneously but who is unconscious It prevents the tongue from falling back against the posterior pharynx and obstructing the airway It allows healthcare providers to suction secretions

Nasopharyngeal airway

Is inserted through the nares If breathing is ineffective or absent, bag-valve-mask ventilation is necessary

Carboxyhemoglobin

Hemoglobin that is bound to carbon monoxide and therefore is unable to bind with oxygen, resulting in hypoxemia

Heat exhaustion manifestation

High body temp Headaches Anxiety Syncope Profuse diaphoresis Goose flesh Orthostasis

Potential urethral injury is manifested by

High riding prostate gland discovered during a rectal examination

Emergent patients

Highest priority

Frost bite

Is trauma from exposure to freezing temperatures and freezing of the intracellular fluid and fluids in intracellular spaces Frozen extremity may be hard, cold, and insensitive to touch and may appear white, or mottled blue-white It results in cellular and vascular damage Can result in venous stasis and thrombosis Body parts most affected- feet, hands, nose, and ears It ranges from first degree (redness and erythema) to fourth degree (full deep tissue destruction)

Replacement fluids may include

Isotonic electrolyte solutions (lactated Ringers solution, normal saline) Colloids Blood component therapy

A hyperbaric oxygen chamber may be used to

Hyperoxygenate crushed tissue if indicated

Complications of nonfatal drowning

Hypoxia Acidosis

Satisfactory ventilation may prevent

Hypoxia Hypercapnia

Multiple trauma

Immediately after injury the body is hypermetabolic, hypercoagulable, and severely stressed

Management of crush injuries

In conjunction with maintaining ABC the pt is observed for acute kidney injury

Grief

Is a complex emotional response to anticipated or actual loss The key nursing intervention is to help family members work through their grief and to support their coping mechanisms, letting them know that it is normal and acceptable for them to cry, feel pain, and express loss Hospital chaplain and social services staff serve as invaluable members of the team when assisting families to work through their grief

Hypothermia

Is a condition in which the core (internal) temperature is 35C or less as a result of exposure to cold or an inability to maintain body temp in the absence of low ambient temperatures Urban hypothermia (extreme exposure to cold in an urban setting) is associated with high mortality rate Alcoholic ingestion increases susceptibility because it causes systemic vasodilation Hypothyroidism and phenothiazines decrease the ability to shiver Fatigue and sleep deprivation - associated with the development of hypothermia Wet clothing accelerates heat loss and immersion in cold water increases heat loss by 25% Cold fluids, unharmed oxygen, exposure during examination If pt has frostbite, hypothermia takes precedence in tx

Obesity considerations

It is generally more challenging to insert IV lines and airways. Ventilation can be a challenge from the increased weight of the chest wall and the increased incidence of hypoventilation and sleep apnea Special consideration must be taken with lipophilic meds, as they take longer to clear from the larger volume of adipose tissue Complications common with hospital stay: respiratory failure, acute kidney injury, pneumonia, DVT, and pressure ulcers ARDS and sepsis Preventative measures: encourage turning, coughing, and deep breathing exercises to prevent atelectasis Early backboard removal and providing early preventive respiratory care, are targeted to improve recovery time

Endotracheal intubation

Its purpose is to establish and maintain airway in pts with respiratory insufficiency or hypoxia It is indicated to establish an airway for a pt who cannot be adequately ventilated with oropharyngeal or nasopharyngeal airway, bypass an upper airway obstruction, prevent aspiration, permit connection of pt to a resuscitation bag or mechanical ventilator, or facilitate the removal of tracheobronchial secretions

Management of nonfatal drowning

Maintain cerebral perfusion and adequate oxygenation to prevent further damage to vital organs Cardiopulmonary resuscitation is the factor with the greatest influence on survival Endotracheal tube with PEEP improves oxygenation, prevents aspiration, and corrects intrapulmonary shunting and ventilation-perfusion abnormalities - if pt does not breath spontaneously Monitoring ABGs Nasogastric tube to decompress the stomach and to prevent the pt from aspirating gastric contents Rewarming for hypothermia

Nurses in the ED are usually fitted with

Personal high efficiency particulate air (HEPA) filter masks to use when treating pts with airborne diseases

Management of airway obstruction

Partial obstruction - if a pt can breath and cough spontaneously - encourage forceful cough Weak, ineffective cough, high pitched noise while inhaling, increased respiratory difficulty, or cyanosis - the pt should be managed as if they were complete airway obstruction After the obstruction is removed , rescue breathing is initiated If the pt has no pulse, cardiac compressions are instituted to provide oxygen to the brain, heart, and other vital organs until definitive medical treatment can restore and support normal heart and ventilatory activity

Abdominal CT scans

Permit detailed evaluation of abdominal contents and retroperitoneal examination

A nurse is performing triage at the scene of a building collapse and is using a five-level triage system. Place the categories below in the proper order from most to least immediate. 1Resuscitation 2Urgent 3Minor 4Nonurgent 5Emergent

Resuscitation Emergent Urgent Nonurgent Minor The five-level system of triage classifies patients as follows: resuscitation (need immediate treatment to prevent death); emergent (may deteriorate rapidly and develop a major life-threatening situation or require time-sensitive treatment); urgent(need two or more resources to provide care and conditions are not life-threatening); nonurgent (need only one resource for needs and condition is not life-threatening); and minor (require no resources for care with no life-threatening condition).

Management of heat induced illnesses

Stabilizing oxygenation using the CABs ( circulation, airway, breathing) formerly known as the ABCs of basic life support- this includes establishing IV access for fluid administration Removal of clothes reduces temp 39C (102F) as rapidly as possible preferably within 1hr Cool sheets and towels or continuous sponging with cool water Ice applied to the neck, groin, chest, and axillae while spraying with tepid water Cooling blankets Immersion of the pt in a cold water bath is the optimal method of cooling (if available) Temp monitored with a thermistor placed in the rectum, bladder, or esophagus to evaluate core temp The cooling process should stop at 38C (100.4) in order to avoid iatrogenic hypothermia Throughout treatment, the patient's status is monitored carefully, including vital signs, ECG findings (for possible myocardial ischemia, myocardial infarction, and dysrhythmias), central venous pressure (CVP), and level of responsiveness A seizure may be followed by recurrence of hyperthermia 100% oxygen given Endotracheal intubation and mechanical ventilation to support failing cardiopulmonary systems may be required IV infusion therapy of normal saline or lactated Ringer solution is initiated as directed for fluid replacement and maintainance of adequate circulation Blood specimen to detect blood disorders such as (DIC) disseminated intravascular coagulation Dialysis for AKI Anticonvulsants Potassium for hypokalemia Sodium bicarbonate to correct metabolic acidosis Benzodiazepines such as diazepam for seizures Phenothiazine such as Thorazine to suppress shivering

Helping family members cope with sudden death

Take the family to a private place Talk to the family together so that they can grieve together and hear information given together Reassure the family that everything possible was done; inform them the treatment rendered Avoid using euphemisms such as "passed on". Sho the family you care by touching, offering coffee, water, and the services of a chaplain Encourage family members to support each other and to express emotions freely (grief, loss, anger, helplessness,tears, disbelief) Avoid giving sedation to family members; this may mask or delay the grieving process, which is necessary to achieve emotional equilibrium and to prevent prolonged depression Encourage the family to view the body if they wish; this action helps integrate the loss. Cover disfigured and injured areas before the family sees the body. Go with the family and do not leave them alone. Show acceptance by touching the body to give the family "permission" to touch Spend time with the family, listening to them and identifying any needs that they may have for which the nursing staff can be helpful Allow family members to talk about the deceased and what he or she meant to them; this permits ventilation of feelings of loss. Encourage the family to talk about events preceding admission to the emergency department. Do not challenge initial feelings of anger or denial Avoid volunteering unnecessary information ( the pt was drinking)

Splinting immobilizes

The joint at a site distal and proximal to the fracture, relieves pain, restores or improves circulation, prevents further tissue injury and prevents a closed fracture for support Upper extremities must be splinted in a functional position After splinting, the vascular status of the extremity is checked by assessing color, temperature, pulse, and blanching of the nailbed

Multiple trauma

Trauma caused by a single catastrophic event that causes life threatening injuries to at least two distinct organs or organ systems

One of the first principles of emergency care is

Triage

Fluid replacement

Two large gauge IV catheters are inserted, preferably in an uninjured extremity to provide a means for fluid and blood placement

Secondary survey

an assessment of the patient triaged to the emergent or resuscitation category that commences after the primary survey is completed and life-threatening insults addressed; includes obtaining vital signs, completing a head-to-toe examination, and obtaining the patient's pertinent medical-surgical history, including the history of the current event

Antivenin

antitoxin manufactured from venom of poisonous snakes to assist the pt's immune system response to an envenomation


Set pelajaran terkait

Revocation, Suspension. and Cancellation

View Set

E-Commerce- 8.04 JavaScript Questions

View Set

Chapter 4: The Great Lakes- The US and Canada'sFreshwater Treasures

View Set

Disease and Conditions Medical Terminology

View Set